You are on page 1of 48

Practice Questions 1

A1 C1 3
1. A forty-eight year-old female sustained a fracture to her left shoulder. Treatment is proceeding well except that
with left shoulder flexion you notice the scapula protract and elevate early and it continues to move excessively.
Physical therapy intervention should emphasize:
A. glenohumeral mobilization and strengthening of scapular stabilizers to regain normal scapulohumeral
movement.
B. glenohumeral mobilization, and strengthening of the rotator cuff muscles to regain muscle balance.
C. scapulothoracic mobilization and strengthening of the pectoralis major and minor muscles to regain normal
scapulohumeral rhythm.
D. stretching of scapular stabilizers and strengthening of the pectoralis major and minor muscles to regain muscle
balance.

A2 C8 1
2. A 59 year-old ex-machinist demonstrates significant age-related hearing loss, presbycusis. When trying to
communicate with this patient you would NOT suspect:
A. bilateral hearing loss, at all frequencies since he has had this problem for a number of years.
B. decreased language comprehension.
C. poor auditory discrimination.
D. unilateral hearing loss.

A3 C3 3
3. You are supervising the exercise of cardiac rehabilitation outpatient class on a very hot day, with temperatures
expected to be above 90 degrees F. Your class is scheduled for 2 p.m. and the facility is not air conditioned. The
strategy that is unacceptable is to:
A. change the time of the exercise class to early morning or evening.
B. decrease the exercise intensity by slowing the pace of exercise.
C. increase the warm-up and cool-down periods to equal the total aerobic interval in time.
D. make the exercise intermittent by adding rest cycles.

A4 C6 1
4. A patient has a transtibial amputation and has recently been fitted with a PTB socket. During your initial prosthetic
checkout, you instruct the patient to walk several times in the parallel bars. You then have him sit down and take
the prosthesis off. You inspect the skin. You would expect no redness in the area of the:
A. anterior tibia, tibial crest, and fibular head.
B. patellar tendon and tibial tuberosity.
C. medial tibial and fibular plateaus.
D. distal end of the residual limb.

A5 C4 2
5. A patient who is terminally ill with cancer is in tears, unable to cope with the changes in her life and with her
current hospitalization. You have a referral for gait training so she can be discharged to home under hospice care.
The BEST approach for you to take is:
A. ask the patient questions so you can gain a detailed history.
B. encourage denial so she can cope better with her lifes challenges.
C. ignore her tears and focus on her therapy but in a compassionate manner.
D. take time now to allow the patient to express her fears and frustrations.

A6 C5 3
6. Your examination reveals muscle spasms of the deep hip rotators, which are compressing the sciatic nerve and
producing pain in the posterior hip region. The MOST effective setting of ultrasound in this case is:
A. 1 MHz continuous at 1.0 W/cm2.
B. 1 MHz pulsed at 1.0 W/cm2.
C. 3 MHz continuous at 1.0 W/cm2.
D. 3 MHz pulsed at 1.0 W/cm2.

05 Sullivan A
2 Practice Questions

A7 C2 2
7. A 17 year-old patient is recovering from a complete spinal cord injury, at the level of L2. The expected outcome in
this case would MOST likely include:
A. a spastic or reflex bladder.
B. greater loss of arm function than leg function with early loss of pain & temperature sensation.
C. loss of motor function, pain & temperature sensation below the level of the lesion with light touch,
proprioception and position sense preserved.
D. some recovery of function since damage is to peripheral nerve roots.

A8 C7 3
8. A physical therapist is gait training a patient with left hemiplegia. The patients new AFO arrives, but the therapist
is overwhelmed with too many patients and asks the physical therapy student to take over. This is the students
first affiliation (second day) and she has never performed an orthotic checkout for a patient with an AFO. The
supervising therapist will be in the same vicinity treating other patients. This task should:
A. be considered an advanced task, but allowable for the student to perform as a good learning experience.
B. be designated as a more advanced task and more appropriately delegated to another physical therapist.
C. be designated as a routine task and appropriate for the student who could call out to the supervisor if problems
arose.
D. not be completed now and the patient sent back to his room.

A9 C1 2
9. A patient has been diagnosed with acute synovitis of the temporomandibular joint. Early intervention should focus
on:
A. application of an intraoral appliance and phonophoresis.
B. instruction to eat a soft food diet and phonophoresis.
C. joint mobilization and postural awareness.
D. temporalis stretching and joint mobilization.

A10 C1 3
10. During a cervical spine examination you observe restricted left rotation of the C7-T1 spinal level. After stabilizing
the thoracic spine, your hand placement for mobilization to improve left rotation should be at the:
A. posterior left C6 articular pillar.
B. posterior left C7 articular pillar.
C. posterior right C7 articular pillar.
D. T1 spinous process.

A11 C8 1
11. An appropriate fine motor behavior that should be established by 9 months of age would be the ability to:
A. build a tower of 4 blocks.
B. hold a cup by the handle while drinking.
C. pick up a raisin with a fine pincer grasp.
D. transfer objects from one hand to another.

A12 C2 1
12. A patient with a 10 year history of Parkinsons disease has been taking L-dopa for the last 5 years. He presents in
your clinic with deteriorating function. He is not longer able to transfer or walk independently. During the oral
interview, you observe facial grimacing with twitching of the lips and tongue protrusion. He appears restless, with
constant dancing, athetoid-like movements of his legs. Your BEST course of action is to:
A. complete your evaluation focusing specifically on his main problems of rigidity and bradykinesia.
B. document your observations and refer him back to his physician for evaluation of possible dopamine toxicity.
C. talk to his wife to see if he is taking any drugs with hallucinogenic effects.
D. use isokinetic dynamometry to assess his inability to generate torque output during fast movements.

05 Sullivan A
Practice Questions 3

A13 C7 3
13. A patient who is to undergo surgery for a chronic shoulder dislocation asks you to explain the rehabilitation
following a surgical reconstructive procedure he is scheduled to undergo. Your BEST response is to:
A. explain how patients typically respond to the surgery and outline the progression of exercises.
B. explain in detail about the surgical procedure.
C. refer the patient to a physical therapy clinical specialist who is an expert of shoulder reconstructive
rehabilitation.
D. tell the patient to ask his surgeon for this information.

A14 C3 2
14. Which of the following is NOT an appropriate reason to terminate a maximum exercise tolerance test for a patient
with pulmonary dysfunction?
A. ECG monitoring reveals diagnostic ischemia.
B. PaO2 decreases 20 mmHg.
C. patient reaches age-predicted maximal heart rate.
D. patient states he is maximally short of breath.

A15 C6 3
15. A patient with a complete T10 paraplegia is receiving his initial ambulation training. He has received bilateral
Craig-Scott knee-ankle-foot orthoses and is being trained with axillary crutches. Since a reciprocal gait pattern is
problematic for him, the BEST initial gait pattern to teach him is:
A. four-point.
B. swing-through.
C. swing-to.
D. two-point.

A16 C4 2
16. After mastectomy, a patient cannot accept the loss of her breast. She reports being weepy all the time with loss of
sleep. She is constantly tired and has no energy to do anything. The BEST action you can take is:
A. observe her closely for possible suicide.
B. request her primary physician to refer her for psychological evaluation.
C. tell her shes over-reacting, she has to get on with her therapy.
D. tell the nurse case manager to monitor her behavior.

A17 C5 3
17. A 23 year-old college volleyball player complains of moderate pain resulting from a left hamstring strain four
weeks ago. The focal point of pain and tightness is noted where a hematoma developed initially. The specific
massage technique that would be MOST beneficial in this case is:
A. friction.
B. kneading.
C. stroking.
D. tapotement.

A18 C8 2
18. A six year-old boy born with myelomeningocele at the L2 level is referred for physical therapy treatment at home.
In determining the physical therapy plan of care, it would NOT be appropriate to emphasize:
A. gait training with a reciprocating gait orthosis.
B. transfer training from floor to wheelchair.
C. upper extremity strengthening with weights.
D. vigorous range of motion of the lower extremities.

A19 C1 1
19. During your examination of a patient who complains of back pain you found pain with end range AROM into hip
flexion, abduction and external/lateral rotation. The structure most likely causing the patients pain is the:
A. hamstring muscle.
B. hip joint.
C. piriformis muscle.
D. SI joint.

05 Sullivan A
4 Practice Questions

A20 C7 2
20. A therapist wants to compare frequencies of carpal tunnel syndrome occurring in different groups of individuals:
assembly line workers and computer programmers. The MOST appropriate statistical tool to use for analysis of the
data is:
A. chi square test.
B. normal distribution curve.
C. simple one-way ANOVA.
D. t test.

A21 C7 1
21. In a research study in which there is a skewed distribution with extreme scores on a balance measure that deviate
from the performance of the total group, the MOST accurate representation of central tendency is:
A. mean.
B. median.
C. mode.
D. standard deviation

A22 C2 1
22. A 54 year-old factory worker injured his right arm in a factory press with damage to the ulnar nerve at the elbow. A
diagnostic EMG was performed with evidence of spontaneous fibrillation potentials. In this case, the physical
therapy plan of care should consider that:
A. axonotmesis is occurring.
B. denervation atrophy has occurred.
C. reinnervation is complete.
D. reinnervation is in process.

A23 C6 3
23. A 67 year-old patient is recovering from a left CVA resulting in severe right hemiplegia. Additionally, he has a
large diabetic ulcer on his left foot with pitting edema, requiring elevation of that extremity. The MOST
appropriate wheelchair prescription for this patient would be a:
A. hemiplegic chair.
B. lightweight active duty wheelchair.
C. one-arm drive chair.
D. powered wheelchair with joystick.

A24 C3 1
24. A 14 year-old boy with advanced Duchenne muscular dystrophy is administered a pulmonary function test. The
value that is UNLIKELY to show any deviation from normal is:
A. FEV1.
B. functional residual capacity.
C. total lung capacity.
D. vital capacity.

A25 C6 3
25. A 29 year-old woman fractured her right midtibia in a skiing accident three months ago. After cast removal, a
severe foot drop was noted. The patient desires to try electrical stimulation orthotic substitution. You would set up
the functional electrical stimulation to contract the appropriate muscles during:
A. foot flat.
B. push off.
C. swing phase.
D. toe off.

A26 C4 2
26. You have a small area of dermatitis on the back of your hand with moderate exudate. You are scheduled to treat a
patient with HIV for management of an open wound. You should:
A. continue with treatment as scheduled but wash your hands thoroughly before and after.
B. double glove and treat as scheduled.
C. refuse to treat that patient.
D. use sterile precautions with mask and gloves.

05 Sullivan A
Practice Questions 5

A27 C1 2
27. You are a home therapist treating a patient who underwent a total hip replacement four weeks ago. You notice that
the patient arches his lumbar spine in supine. He states that it is uncomfortable and doesnt remember having the
problem before. The patient is unable to maintain a comfortable supine position due to:
A. poor abdominal strength.
B. tight hamstrings muscles.
C. tight iliopsoas muscle.
D. tight piriformis muscle.

A28 C8 3
28. Your 102 year-old patient has been hospitalized for the past three days with an undisclosed ailment. After running
numerous tests and finding nothing to explain her increasing weakness and fatigue, the physicians are being
pressured to discharge her tomorrow. She lives alone in a first floor apartment. You have determined her
ambulation endurance to be only up to 15 feet, not enough to allow her to get from her bed to the bathroom (a
distance of 20 feet). You recommend:
A. a live-in nurse (24 hour coverage) until her condition improves.
B. a skilled nursing facility placement until her endurance increases.
C. environmental changes, a bedside commode, and referral for home health services.
D. postponing her discharge until she can complete the needed 20 feet.

A29 C2 3
29. You have determined a patient with a right CVA that you are currently treating has a profound deficit of
homonymous hemianopsia. The BEST initial strategy to assist the patient in compensating for this deficit is to:
A. make the patient aware of his deficit and teach him to turn his head to the affected left side.
B. place items, eating utensils on his left side.
C. provide constant reminders, printed notes on his left side, telling him to look to the left.
D. rearrange his room so while in his bed his left side is facing the doorway.

A30 C2 2
30. A patient with multiple sclerosis is agitated, irritated, and tired during treatment. Your BEST response as the P.T. is
to:
A. begin pool therapy to promote relaxation.
B. ignore these behaviors and continue treating.
C. reduce the exercise intensity and provide relaxation strategies.
D. treat the patient in a cool environment.

A31 C7 3
31. A 72 year-old medically stable individual requires custodial care in the home. She is severely disabled with
rheumatoid arthritis and is in a great deal of pain. She presents with significant deformities which limit her
functional abilities and is dependent in all basic activities of daily living. A recent exacerbation of her disease has
left her bed-bound for the past 2 weeks. Appropriate physical therapy services would be covered by:
A. Medicaid.
B. Medicare.
C. Medigap policies.
D. only by HMO or private insurance policies.

A32 C3 2
32. A patient with a purulent venous insufficiency ulcer near the left medial malleolus is seen at home by a physical
therapist. The MOST important goal/intervention for the therapist to try to achieve with this patient is:
A. daily changes of elastic wraps to reduce fluid buildup.
B. daily warm water baths to improve circulation.
C. increase ambulation endurance to hasten wound healing.
D. instruct in proper dressing changes and wound care.

05 Sullivan A
6 Practice Questions

A33 C6 1
33. Your patient is having difficulty bearing weight on the left leg. She is unable to advance the tibia forward and
abbreviates the end of the stance phase on the left going directly into swing phase. The MOST likely cause of her
problem is:
A. hip extensor weakness.
B. spasticity of the anterior tibialis muscle.
C. spasticity or contracture of the plantarflexors.
D. weakness or contracture of the dorsiflexors.

A34 C1 1
34. With a traction injury to the anterior division of the brachial plexus you would expect to see weakness of the elbow
flexors, wrist flexors and forearm pronators. You would also expect to find additional weakness in:
A. forearm supination.
B. lateral rotation of the shoulder.
C. thumb abduction.
D. wrist extension.

A35 C8 3
35. A 95 year-old has recently been admitted to your skilled nursing facility following a fall-related injury (fractured
hip with open reduction, internal fixation). Since she lived alone on the second floor, she was unable to return
home. She is extremely agitated over her placement here and demonstrates early signs of dementia. She tells you
leave me alone, I just want to get out of here! An important approach to take while working with this client is to:
A. be calm and supportive, using only one or two level commands.
B. establish the rules: tell her exactly what she is to do while you are with her.
C. use gesture or sign language to communicate with her.
D. promise her anything as long as she gets up and walks for you.

A36 C8 1
36. A 72 year-old woman is being treated for depression following the death of her husband. She is currently taking
antidepressant medication (tricyclics) and has a recent history of a fall. You suspect the precipitating cause of the
fall can be attributed to side effects of her medication resulting in:
A. cardiac arrhythmias
B. dyspnea.
C. hyperalertness.
D. postural hypotension.

A37 C3 1
37. An 82 year-old patient and his caregivers should understand the common side effects of the medication that he is
taking. He is continually in and out of congestive heart failure and has been taking digitalis (Digoxin) to improve
his heart function. You will know he and his caregivers understand the adverse side effects of this medication if
they tell you they will contact the patients physician if he demonstrates:
A. confusion and memory loss.
B. involuntary movements and shaking.
C. slowed heart rate.
D. weakness and palpitations.

A38 C7 3
38. The Director of Physical Therapy from a large teaching hospital is asked to develop an operating budget for the
upcoming fiscal year. The item that would NOT be included in an operating budget is:
A. a treadmill purchase.
B. equipment maintenance.
C. housekeeping supplies.
D. long distance telephone calls.

05 Sullivan A
Practice Questions 7

A39 C4 1
39. You are treating a patient with active hepatitis B infection. Transmission of the disease is best minimized if you:
A. avoid direct contact with any part of the patient.
B. have the patient wear a gown and mask every time your are in the room.
C. have the patient wear gloves to prevent him from touching you.
D. wear gloves if there is direct contact with blood or body fluids.

A40 C5 3
40. A wrestler complains of pain (7/10) and limited range of motion of the right shoulder as a result of chronic
overuse. You elect to use procaine hydrochloride iontophoresis as part of your intervention for this patients
problems. To administer this substance, it would be appropriate to use:
A. continuous biphasic current with the medication under the anode.
B. continuous monophasic current with the medication under the anode.
C. continuous monophasic current with the medication under the cathode.
D. interrupted monophasic current with the medication under the cathode.

A41 C1 2
41. A contraindication to initiating joint mobilization on a patient with chronic pulmonary disease may include:
A. concurrent inhalation therapy.
B. functional chest wall immobility.
C. long term corticosteroid therapy.
D. reflex muscle guarding.

A42 C8 3
42. Long term care for institutionalized elderly who have reduced their financial resources or qualify for low-income
status is typically funded by:
A. Health Maintenance Organizations.
B. Medicaid.
C. Medicare.
D. Social Security Administration.

A43 C3 1
43. A 45 year-old computer programmer, with no significant past medical history, presents to the emergency room
with complaints of fever, shaking chills and a worsening productive cough. He has chest pain over the posterior
base of his left thorax which is made worse on inspiration. An anterior-posterior X-ray shows an infiltrate on the
lower left thorax at the posterior base. This patients chest pain is MOST likely caused by:
A. angina.
B. infected pleura.
C. inflamed tracheobronchial tree.
D. trauma to the chest.

A44 C7 3
44. Equipment safety is essential in all physical therapy clinics. Regularly scheduling equipment maintenance
programs to ensure that all equipment is calibrated, lubricated, and adjusted according to manufacturers
guidelines is an important element for patient and staff safety. All of the following procedures should be followed
to ensure safety EXCEPT:
A. conducting educational sessions for staff regarding the indications and contraindications for all equipment.
B. documenting all preventive maintenance and keeping this information on file.
C. supervising new staff and students in the use of all newly purchased equipment.
D. training all staff to do simple repairs on all electrical equipment if a breakdown should occur.

A45 C4 1
45. A 92 year-old woman presents with hot, red, and edematous skin over the shins of both lower extremities. She also
has a mild fever. The MOST likely cause of her symptoms is:
A. cellulitis.
B. dermatitis.
C. herpes simplex infection.
D. scleroderma.

05 Sullivan A
8 Practice Questions

A46 C5 2
46. A 10 year-old presents with pain and limited ROM following surgical repair of the medial collateral ligament and
anterior cruciate ligaments. The modality that would be CONTRAINDICATED in this case is:
A. interferential current.
B. shortwave diathermy.
C. transcutaneous electrical stimulation.
D. ultrasound.

A47 C6 3
47. Hypertrophy is the muscular response to strength training. This can be expected to occur following at least:
A. 1-2 weeks of training.
B. 2-3 weeks of training.
C. 3-4 weeks of training.
D. 6-8 weeks of training.

A48 C1 3
48. A diagnosis of bicipital tendinitis has been made following an evaluation of a patient with shoulder pain. The
BEST shoulder position to expose the tendon of the long head of the biceps for application of phonophoresis
would be:
A. abduction.
B. external/lateral rotation and extension.
C. horizontal adduction.
D. internal/medial rotation and abduction.

A49 C7 3
49. A patient is unable to bring her foot up on the next step during a training session on stair climbing. Your BEST
course of action to promote active learning is to have the patient:
A. balance on the stairs while you passively bring the foot up.
B. practice marching in place in the parallel bars.
C. practice standing-up from half-kneeling.
D. step up onto a low step while in the parallel bars.

A50 C2 3
50. A 76 year-old patient exhibits impaired balance. A diagnostic work-up has failed to reveal any specific etiology.
An initial intervention for this patient would NOT include:
A. limits of stability re-education including postural sway training.
B. practice in maintenance of a wide base of support during gait and turns.
C. sit-to-stand and stand-to-sit activity training.
D. tandem walking and single limb stance.

A51 C6 3
51. A 38 year-old patient with spinal cord injury is being discharged home after a 2 month course of rehabilitation. In
preparation for discharge, you and your team visit the home and find he has 3 standard height steps going into his
home. A ramp will have to be constructed for his wheelchair. The recommended length of his ramp should be:
A. 120 inches (10 feet).
B. 192 inches (16 feet).
C. 252 inches (21 feet).
D. 60 inches (5 feet).

A52 C3 1
52. Following pneumonectomy, an expected change in a patients status would NOT include:
A. asymmetrical breathing.
B. decreased breath sounds.
C. deviated trachea.
D. increased tidal volume.

05 Sullivan A
Practice Questions 9

A53 C3 1
53. An older person with diagnosis of congestive heart failure (CHF) should recognize the symptoms of exertional
intolerance. You will know this client is properly informed if she can tell you these symptoms are:
A. dizziness, visual blurring especially with turns and quick movements.
B. overwhelming weakness with difficulty in standing up and walking.
C. severe, uncomfortable chest pain with shortness of breath.
D. shortness of breath at rest and with limited activity, and sudden weight gain

A54 C6 3
54. You are instructing a physical therapy student in proper positioning to prevent the typical contractures in the
patient with a transtibial amputation. You stress:
A. maximize out-of-bed time with sitting in a chair.
B. position in prone lying and sitting with full knee extension.
C. position in prone-lying with slight knee flexion.
D. position in supine with a small pillow under the knee.

A55 C4 2
55. You receive a referral to ambulate a patient who is insulin dependent. In a review of her medical record, you notice
her blood glucose level for that day is 310 mg/dL. Your BEST course of action is to:
A. ambulate the patient as planned but monitor closely.
B. postpone therapy and coordinate with the nurse regarding insulin management and exercise.
C. refrain from ambulating the patient, reschedule for tomorrow.
D. talk to the nurse about seeing the patient later on that day.

A56 C8 3
56. A physical therapy plan of care for a newborn with Erb-Klumpke Palsy would NOT include:
A. age appropriate movements of the upper extremity.
B. gentle ROM after immobilization.
C. partial immobilization of limb across abdomen.
D. splinting the shoulder in abduction and internal rotation.

A57 C1 3
57. A six month-old child was referred to physical therapy for right torticollis. The MOST effective method to stretch
the muscle is by positioning the head and neck into:
A. extension, left side-bending, and right rotation.
B. extension, right side-bending, and left rotation.
C. flexion, left side-bending, and left rotation.
D. flexion, right side-bending, and left rotation.

A58 C7 3
58. A student you are supervising is on final affiliation following completion of academic training. He is overheard
discussing a patients history in the elevator. When you later point this out to the student, he tells you he was
unaware of any hospital policy regarding confidentiality. Your BEST analysis of this situation is that:
A. compliance was not a realistic expectation since he just arrived at this facility.
B. he should be expected to value patient confidentiality.
C. now that he is aware of confidentiality restrictions he should do better the next time.
D. since this is not strictly part of the professional code of ethics you should not reasonably expect him to
demonstrate adherence to this concept.

A59 C7 2
59. A therapist is performing clinical research in which a specific myofascial technique is applied to a patient with
chronic back pain. She is using a single case experimental design with an A-B-A-B format. Her research
hypothesis states that pain rating scores will decrease with the treatment intervention. Acceptance of this
hypothesis would be indicated if:
A. B is equal to A.
B. B is greater than A, at the .05 level.
C. B is greater than A, at the 1.0 level.
D. B is less than A.

05 Sullivan A
10 Practice Questions

A60 C2 3
60. Isokinetic training can best be used in the rehabilitation of patients with stroke during the late stages of recovery
to:
A. improve initiation of movement.
B. improve rate control at faster movement speeds.
C. improve rate control at slower movement speeds.
D. increase strength of synergy components.

A61 C6 3
61. A 72 year-old patient with a transfemoral amputation is unable to wrap his residual limb. Your BEST course of
action is to:
A. apply a temporary prosthesis immediately.
B. consult with the vascular surgeon about the application of an Unnas paste dressing.
C. redouble efforts to teach proper wrapping.
D. use a shrinker.

A62 C3 2
62. A 75 year-old patient with peripheral vascular disease has been referred for conditioning exercise. He demonstrates
moderate claudication pain in both legs following a 12 minute walking test. The MOST appropriate exercise
frequency and duration for this patient is:
A. 2 times/week, BID 20 minutes/session.
B. 3 times/week, 30 minutes/session.
C. 3 times/week, 60 minutes/session.
D. 5 times/week, BID 10 minutes/session.

A63 C1 1
63. The radiographic view shown in the diagram that demonstrates the observed spinal defect is: Twomey L, Taylor J
(2000) Physical Therapy of the Low Back, 3rd ed. Philadelphia, Churchill Livingstone, Figure 7-1, page 204, with
permission.
A. frontal.
B. lateral.
C. oblique.
D. posterolateral.

A64 C4 2
64. A physical therapist is working with a patient with metastatic breast cancer who has been told that she has only
months to live. She is quite angry and disruptive during therapy. What is the MOST appropriate intervention for
this patient?
A. allow the patient to express her anger while refocusing her on effective coping strategies.
B. forbid all expressions of anger as she is only hurting herself.
C. provide honest, accurate information about her illness and rehab plan of care.
D. provide opportunities for the patient to question her impending death but limit all expressions of anger.

A65 C1 1
65. A forty year-old female cafeteria worker sustained a right-sided injury to her back while playing golf. She was
driving the ball when she felt an immediate sharp pain in her right low back. She states that in the morning she is
stiff and her pain eases after taking a shower. Based on the above information, the structure MOST likely involved
is:
A. a disc.
B. a facet joint.
C. a nerve root.
D. the arterioles which supply circulation to the spinal cord.

05 Sullivan A
Practice Questions 11

A66 C7 3
66. At 10 a.m. a physical therapist working on a spinal cord unit is treating a patient with paraplegia at the T3 level.
The therapist smells alcohol on his breath and the patient is having difficulty accomplishing a bed-to-chair transfer
that was previously done without assistance. In this case the physical therapist should:
A. confront the patient and ask if he has been drinking.
B. document and report suspicions of alcoholism to the rehabilitation team at the weekly meeting.
C. document the findings and immediately inform the patients physician about the situation.
D. immediately inform the nurse in charge.

A67 C2 3
67. A patient with right hemiparesis has difficulty clearing the affected foot during the swing phase of gait. An
appropriate physical therapy intervention for the right lower extremity might include:
A. bridging.
B. forward step-ups in standing using graduated height steps.
C. pushing backward while sitting on a rolling stool.
D. sitting on a Swiss ball, alternating lateral side steps and back to neutral.

A68 C6 2
68. A 19 year-old patient has a complete spinal cord injury at the level of L1. His primary goal is to walk again. You
decide it would be MOST appropriate to recommend that this patient use:
A. a reciprocating gait orthosis and walker.
B. a wheelchair, because ambulation is unrealistic.
C. bilateral AFOs and Lofstrand crutches.
D. bilateral KAFOs with thoracolumbosacral extension control.

A69 C3 2
69. A patient is four weeks post myocardial infarction. Resistive exercises using weights are:
A. always contraindicated during acute and post-acute phases of cardiac rehabilitation due to expected elevations
of BP.
B. appropriate if intensities are kept below 40% maximal voluntary contraction.
C. appropriate if intensities are kept below 85% maximal voluntary contraction.
D. safe during all phases of rehabilitation if judicious monitoring of HR is used.

A70 C5 3
70. A patient diagnosed with cervical radiculitis has been referred to you for mechanical traction. You are applying the
traction using the cervical halter for 5 minutes at 20 neck flexion using 10 pounds. The patient complains of pain
in the area of the TMJ. You should:
A. change the angle of pull.
B. decrease the traction poundage.
C. decrease the treatment time.
D. discontinue the treatment.

A71 C4 3
71. On the first day following a cesarean delivery, the physical therapists initial intervention would consist of teaching
the new mother:
A. ankle exercises to prevent thrombophlebitis.
B. assisted ambulation.
C. assisted breathing and coughing and pelvic floor exercises.
D. partial sit-ups and pelvic floor exercises.

A72 C1 2
72. It is important to note the status of the pars interarticularis on the X-ray report. A problem with this part of the
vertebra could possibly lead to:
A. spondylolisthesis with discal herniation.
B. spondylolisthesis with possible anterior slippage of the vertebral body.
C. spondylolysis resulting in early nerve root compression.
D. spondylolysis with early degeneration of the vertebra.

05 Sullivan A
12 Practice Questions

A73 C8 3
73. The most appropriate school physical therapy intervention to use during class for a child with decreased sitting
balance, but normal tone would be to:
A. adapt a desk and wheelchair to provide adequate sitting balance.
B. use a prone-stander.
C. use a sidelyer.
D. use a therapeutic ball to promote sitting balance.

A74 C2 2
74. A patient with Parkinsons disease demonstrates a highly stereotyped gait pattern characterized by impoverished
movement. The intervention that would be LEAST appropriate to use with this patient is:
A. a rolling walker to compensate for impaired balance.
B. postural exercises to promote trunk and head extension in sitting and standing.
C. range of motion exercises to reduce hip and knee flexion contractures.
D. sidelying, upper and lower trunk rotation segmental patterns using the technique of rhythmic initiation.

A75 C2 1
75. Your patient is 72 and recovering from a right CVA. She tells you she is thirsty and asks you for a can of soda.
When you give her the can and instruct her to open it, she is unable to complete the task. Later after the treatment
session when she is alone you observe her drinking from the can. You suspect she may have a primary deficit in:
A. anosognosia.
B. ideational apraxia.
C. ideomotor apraxia.
D. unilateral neglect.

A76 C7 3
76. Peer review is an important professional activity. Recently, physical therapists have been the focus of vigorous
peer review due to increasing financial pressure imposed by third party payers. An inappropriate use of peer
review is to determine whether care:
A. should be paid for by a third party payer.
B. was appropriate and required the skill of a physical therapist.
C. was cost effective.
D. was provided by the appropriate personnel.

A77 C3 1
77. A 62 year-old patient has chronic obstructive pulmonary disease. The MOST likely pulmonary test result would
be:
A. decreased functional residual capacity.
B. decreased residual volume.
C. increased total lung capacity.
D. increased vital capacity.

A78 C6 1
78. A patient with a transfemoral amputation is being fitted with a quadrilateral socket. Areas of pressure tolerance
would be expected over the:
A. distolateral end of femur and ischial seat.
B. gluteals and adductor magnus.
C. ischial tuberosity, gluteals, and lateral sides of residual limb.
D. perineal area and medial side of femur.

A79 C4 2
79. A patient recently diagnosed with fibromyalgia and chronic fatigue immune system dysfunction demonstrates a
loss of interest in all activities and outlets. She is not eating well and is having problems sleeping. Recently she has
talked about suicide as her only hope. Your BEST course of action is to:
A. call her physician, saying that you cannot do anything for this patient until her psychological outlook is better.
B. discuss her need for medications with her husband.
C. immediately contact her primary physician.
D. present a positive attitude and tell her she will feel better soon.

05 Sullivan A
Practice Questions 13

A80 C6 3
80. A 77 year-old patient has been confined to bed for a period of 2 months and now demonstrates limited ROM in
both lower extremities. Range in hip flexion is 5 to 115 and knee flexion is 10 to 120. The MOST appropriate
intervention to improve flexibility and ready this patient for standing is:
A. dynamic lower extremity splints, applied for 2 hours daily.
B. hold-relax techniques followed by passive ROM 3 times a week.
C. manual passive stretching, 5 repetitions each joint, 2 times a day.
D. mechanical stretching using traction and 5 lb. weights, 2 hours, twice daily.

A81 C1 2
81. A seventy year-old male retired carpenter who has had long term lumbar pain with a previous diagnosis of
degenerative joint disease (DJD) of his lumbar facet joints. He complains of numbness, paresthesias and weakness
of his bilateral lower extremities which increase with extended positions or walking greater than 100 feet. His pain
persists for hours after assuming a resting position. He reports he can ride his stationary bike for 30 minutes
without any problems. Primary physical therapy intervention should include:
A. abdominal and back extension strengthening as the result of spondylolysis.
B. increasing cardiovascular endurance as the result of degenerative arthritis.
C. stretching and limiting extended spinal positions as the result of spinal stenosis.
D. traction and limitation of weight bearing positions as the result of discal dysfunction.

A82 C7 3
82. A physical therapist is instructing a kindergarten teacher in a behavior management program for a child with
developmental disabilities who has been mainstreamed into the regular classroom. The therapist requests that the
teacher encourage the child to maintain a head retracted sitting position in the class. The strategy that would be
MOST helpful in this situation is to:
A. have the teacher encourage the classmates to tell the child to sit up in the chair.
B. have the teacher give a smile sticker when the child sits with head retracted for five minutes.
C. have the teacher issue a verbal reprimand when the child slumps in the chair.
D. train the teacher in manual handling techniques to assist the child in head retraction.

A83 C7 3
83. You have recently attended a professional conference on myofascial release. You wish to share this information
with your colleagues during an inservice session. You BEST initial activity is to:
A. ask your colleagues about their current level of knowledge using a brief questionnaire.
B. determine the best sequence for the learning units within your presentation.
C. provide a comprehensive packet of handouts when you begin the first inservice session.
D. select a suitable time and place for your lecture.

A84 C2 2
84. Following an episode of adhesive capsulitis of the right shoulder, a 52 year-old with a history of left CVA now
exhibits reflex sympathetic dystrophy affecting the right upper extremity. The intervention that should be
AVOIDED in this case is:
A. graduated active exercises.
B. massage to reduce edema.
C. passive manipulation to the shoulder.
D. passive ROM exercises.

A85 C6 3
85. To prepare a patient with an incomplete T12 paraplegia for ambulation with crutches, the upper quadrant muscles
that would be MOST important to strengthen include the:
A. deltoid, coracobrachialis, and brachialis.
B. lower trapezius, latissimus dorsi, and triceps.
C. middle trapezius, latissimus, dorsi and triceps.
D. upper trapezius, rhomboids, and levator scapulae.

05 Sullivan A
14 Practice Questions

A86 C3 2
86. A 72 year-old is recovering at home from a myocardial infarction and percutaneous transluminal coronary
angioplasty. As his home care physical therapist, you decide to use pulse oximetry to monitor his responses to
exercise. An acceptable oxygen saturation rate (SaO2) to maintain throughout the exercise period is:
A. 50%.
B. 82%.
C. 85%.
D. 92%.

A87 C5 2
87. A 42 year-old homemaker presents with acute lateral epicondylitis following participation in a local tennis
tournament. Pain is 9/10. The MOST appropriate initial intervention for this problem is:
A. cold intermittent compression three times per week.
B. cold whirlpool daily until the pain subsides.
C. fluidotherapy three times per week.
D. ice massage B.I.D. until the pain subsides.

A88 C1 1
88. A patient was referred to physical therapy complaining of loss of cervical AROM. His X-rays showed DJD at the
uncinate processes in the cervical spine. The motion that would be MOST restricted would be:
A. extension.
B. flexion.
C. rotation.
D. side-bending.

A89 C1 2
89. A 55 year-old male electrician is unable to pull wire overhead due to a painless inability to reach past 80 degrees
of right shoulder abduction. He has had numerous previous episodes of right shoulder pain over the last ten years
which were diagnosed as shoulder tendinitis. Early subacute physical therapy intervention should focus on:
A. active assistive pulley exercises.
B. gentle grade III translatory glenohumeral mobilizations.
C. modalities to reduce pain and inflammation.
D. resistance exercises for the affected supraspinatus muscle.

A90 C7 2
90. Two therapists are asked to perform a test on the same group of patients using the Functional Independence
Measure (FIM). The results of both sets of measurements reveal differences in therapists scores but not in the
repeat measurements. This is indicative of a problem in:
A. concurrent validity.
B. construct validity.
C. interrater reliability.
D. intrarater reliability.

A91 C2 3
91. Your patient is a 16 year-old recovering from a complete spinal cord injury with C5 quadriplegia. You are
performing PROM exercises on the mat when he complains of a sudden pounding headache and double vision.
You notice he is sweating excessively, and when you take his BP it is 240/95. Your BEST course of action is to:
A. immediately contact the patients physician.
B. lie the patient down immediately, elevate his legs, then call for a nurse.
C. place him in sitting position and continue to monitor BP.
D. sit the patient up, check/empty catheter, and then call for emergency medical assistance.

05 Sullivan A
Practice Questions 15

A92 C6 3
92. A 46 year-old patient presents with a flatfoot deformity with abduction of the forefoot in relation to the weight
bearing line. The forefoot is inverted to the varus position when inspected from the frontal plane. Corrections for
this foot deformity would NOT include a:
A. metatarsal bar.
B. scaphoid pad.
C. Thomas heel.
D. UCBL insert.

A93 C4 3
93. A 10-year old with full thickness burns to both arms is developing hypertrophic scars The BEST intervention to
manage these scars is:
A. custom made pressure garments.
B. occlusive dressings.
C. primary excision followed by autografts.
D. surgical resection (Z-plasty).

A94 C3 1
94. A patient with COPD has developed respiratory acidosis. You instruct the physical therapy student who is
participating in her care to monitor the patient closely for:
A. dizziness or syncope.
B. dyspnea, anxiety, or disorientation.
C. muscle twitching or tetany.
D. tingling or numbness.

A95 C1 2
95. A 34 year-old female in her second trimester of pregnancy was sent to physical therapy with complaints of tingling
and loss of strength in both of her hands. Her symptoms are exacerbated if she is required to use her keyboard at
work for longer than 20 minutes. The MOST appropriate physical therapy intervention would include:
A. dexamethasone phonophoresis to the carpal tunnel.
B. hydrocortisone iontophoresis to the volar surfaces of both wrists.
C. ice packs to the carpal tunnel.
D. placing the wrists in resting splints.

A96 C8 3
96. A child you are working with in the elementary school system has moderate to severe extensor spasticity and
limited head control. The MOST appropriate positioning device would be a:
A. prone stander with abduction wedge.
B. supine stander with abduction wedge.
C. wheelchair with a back wedge and head supports.
D. wheelchair with adductor pommel.

A97 C2 3
97. A patient with complete C6 quadriplegia should be instructed to initially transfer with a sliding board using:
A. pectoral muscles to stabilize elbows, scapular depressors to lift trunk.
B. serratus anterior to elevate trunk with shoulder extensors stabilizing.
C. shoulder extensors, external rotators, and anterior deltoid to position and lock the elbow.
D. triceps, keeping the hands flexed to protect tenodesis grasp.

A98 C7 3
98. You have volunteered to teach a stroke education class on positioning techniques for family members and
caregivers. There will be 12 individuals attending this class, ranging in age from 42 to 82. Your BEST choice of
teaching methods is to utilize:
A. demonstration, practice, and follow-up discussion.
B. lecture with some time for questions at the end of the 30 minute session.
C. multimedia (slides and overheads) to accompany your oral presentation.
D. one on one practice with adequate critiquing of performance.

05 Sullivan A
16 Practice Questions

A99 C3 2
99. A patient with active tuberculosis is referred for physical therapy. Which of the following is NOT an appropriate
precaution?
A. have the patient wear a tight fitting mask while being treated in his room.
B. insure that the patient is in a private, negative pressurized room.
C. wash hands upon entering and leaving the patients room.
D. wear a tight fitting mask while treating the patient.

A100 C1 2
100. A twelve-year old female figure skater has been referred to your clinic for treatment for patellar tendinitis. The
examination reveals that she is unable to hop on the affected lower extremity due to pain. You decide to refer her
back to her pediatrician suggesting that the patient receive an x-ray of her knee. The patient returns for therapy
with the x-ray shown in the figure. Your initial intervention should focus on: Magee D (2002). Orthopedic Physical
Assessment, 4th ed. Philadelphia, W. B. Saunders, Figure 12-149, page 746, with permission.
A. aggressive plyometric exercises with focus on endurance training.
B. fitting patient with crutches for non weight-bearing ambulation and initiation of phonophoresis at 3mHz
continuous wave using hydrocortisone.
C. patient education regarding avoidance of squatting and jumping activities as well as initiation of iontophoresis
using dexamethasone.
D. patient education regarding avoiding falls onto her affected knee and open chain knee extension exercises to
improve quadriceps strength.

A101 C6 2
101. A 40 year-old male with a history of low back pain has been receiving physical therapy for 12 weeks. The patient
is employed as a loading dockworker. He performs repetitive lifting and carrying of boxes weighing between 15
and 30 pounds. An appropriate engineering control to reduce the stresses of lifting and carrying would be to:
A. issue the employee a back support belt.
B. provide a two-wheel handcart for use in moving the boxes.
C. require the worker to attend a class in using correct body mechanics while performing the job.
D. use job rotation.

A102 C1 1
102. Common compensatory postures you would expect for a patient diagnosed with fixed severe forefoot varus are:
A. excessive ankle dorsiflexion and medial rotation of the femur.
B. excessive midtarsal supination and lateral rotation of the tibia.
C. subtalar pronation and medial rotation of the tibia.
D. toeing-in and lateral rotation of the femur.

A103 C1 2
103. A patient has undergone surgery and subsequent immobilization to stabilize the olecranon process. The patient
now exhibits an elbow flexion contracture. In this case, an absolute CONTRAINDICATION for joint mobilization
would be:
A. empty end-feel.
B. firm end-feel.
C. soft end-feel.
D. springy end-feel.

A104 C6 3
104. You observe a physical therapist assistant ambulate a patient for the first time after a left total hip replacement. The
patient is using crutches and is practicing on a level surface. The PTA should guard the patient by standing
slightly:
A. behind and to the intact side, one hand on the gait belt.
B. behind and to the left side, one hand on the gait belt.
C. behind the patient with both hands on the gait belt.
D. in front of the patient, walking backward, with one hand on the gait belt and one hand on the shoulder.

05 Sullivan A
Practice Questions 17

A105 C5 1
105. A 13 year-old severed the median nerve three days ago when his hand went through a glass window. To determine
the motor function of the nerve you perform a chronaxie test. At this time you would expect the chronaxie of the
nerve to be:
A. absent.
B. decreased.
C. increased.
D. unaffected.

A106 C1 3
106. A patient has lumbar spinal stenosis encroaching on the spinal cord. The physical therapist should educate the
patient to AVOID:
A. bicycling.
B. rowing.
C. swimming using a crawl stroke.
D. Tai Chi.

A107 C6 2
107. A patient with post-polio syndrome presents in your clinic with symptoms of myalgia and increasing fatigue. He is
wearing a KAFO which he has had for 10 years. When walking, you observe that he rises up over the sound limb
to advance the orthotic limb forward. Your BEST intervention is to provide:
A. a manual wheelchair with reclining back and elevating legrests.
B. a shoe lift on the orthotic side.
C. a shoe lift on the sound side.
D. an electric wheelchair with joystick.

A108 C5 2
108. Your patient has moderate spasticity of the biceps brachii on the left as a result of a CVA. You choose to use
electrical stimulation to temporarily decrease the effects of hypertonicity in order to work on ADL activities. Your
objective in applying the current is to:
A. fatigue the ipsilateral biceps brachii.
B. stimulate the contralateral biceps brachii.
C. stimulate the contralateral triceps.
D. stimulate the ipsilateral triceps.

A109 C8 2
109. You are working with a four year-old child who has myelodysplasia at the L5 level. At this level the most
appropriate orthosis to recommend for ambulation would be a (an):
A. ankle-foot orthosis.
B. knee-ankle-foot orthosis.
C. parapodium.
D. reciprocating gait orthosis.

A110 C1 2
110. An eleven-year-old male was referred to physical therapy with complaints of vague pain at his right hip and thigh
which radiates to his knee. His AROM is restricted in abduction, flexion, and internal rotation. A gluteus medius
gait was observed with ambulation for 100 feet. Appropriate PT intervention would include:
A. closed-chain partial weight-bearing lower extremity exercises for slipped capital femoral epiphysis.
B. hip joint mobilization to improve the restriction in motion as the result of Legg-Calv Perthes disease.
C. open-chain strengthening of his right hip abductors and internal rotators for avascular necrosis of the hip.
D. orthoses to control lower extremity position as the result of femoral anteversion.

05 Sullivan A
18 Practice Questions

A111 C7 2
111. A researcher states that he expects that there will be no significant difference between 20 and 30 year-olds after a
12 week exercise training program using exercise heart rates and myocardial oxygen consumption as measures of
performance. The kind of hypothesis that is being used in this study is a (an):
A. directional hypothesis.
B. experimental hypothesis.
C. null hypothesis.
D. research hypothesis.

A112 C7 3
112. A physical therapist was treating a patient and the patient in the next bed was uncomfortable and asked the
therapist to move his leg. The therapist placed the leg on 2 pillows as requested by the patient. Unknown to the
therapist this patient had a femoral artery graft 2 days previously. As a result the graft became occluded and the
patient was rushed to surgery for a replacement. The patient claimed the therapist placed his leg too high on the
pillows causing the occlusion of the original graft and sued for malpractice. The hospital administrator decided:
A. it was the patients fault for requesting the position change and therefore supported the action of the physical
therapist.
B. that the physical therapist was functioning according to common protocols of the institution and thus
supported the actions of the therapist.
C. that the therapist was functioning outside the common protocols of the hospital, and therefore did not support
the actions of the physical therapist.
D. to counter-sue the patient because he was responsible for requesting the position change.

A113 C2 3
113. A 26 year-old female has a 3 year history of multiple sclerosis. One of her disabling symptoms is a persistent and
severe diplopia which leaves her frequently nauseated and immobile. An appropriate intervention strategy to assist
her in successfully participating in rehabilitation would be to:
A. give her a soft neck collar to limit head and neck movements.
B. give her special glasses which magnify images.
C. have her close her eyes and practice movements without visual guidance.
D. patch one eye.

A114 C2 3
114. The MOST appropriate positioning strategy for a patient recovering from acute stroke who is in bed and
demonstrates a flaccid upper extremity is:
A. sidelying on the affected side with the affected upper extremity flexed overhead.
B. sidelying on the sound side with the affected upper extremity supported on a pillow with the shoulder
protracted and elbow extended.
C. supine with the affected hand positioned on stomach.
D. supine with the affected upper extremity positioned close to the side of the trunk.

A115 C3 2
115. A patient presents with a large plantar ulcer that will be debrided in the whirlpool. The foot is cold, pale, and
painless. The condition that would most likely result in this clinical presentation is:
A. acute arterial insufficiency.
B. chronic arterial insufficiency.
C. chronic venous insufficiency.
D. deep venous thrombosis.

A116 C5 3
116. A patient has been referred to you following a fracture of the femur six months ago. The cast was removed, but the
patient was unable to volitionally contract the quadriceps. You decide to apply electrical stimulation to the
quadriceps muscle. Your choice of electrode placement and electrical stimulation duty cycle (on:off ratio) would
consist of:
A. large electrodes, closely spaced; 10:30.
B. large electrodes, widely spaced; 10:30.
C. small electrodes, closely spaced; 10:30.
D. small electrodes, widely spaced; 10:10.

05 Sullivan A
Practice Questions 19

A117 C4 2
117. While setting a patient up for cervical traction, you notice a purplish mole with rough edges on the patients neck.
You:
A. call the physician immediately and report your findings.
B. document the skin condition and keep a watchful eye on it.
C. tell the patient if it bleeds at all to report it to his physician.
D. treat the patient but cover the mole with a gauze pad.

A118 C2 1
118. A 15 year-old male suffered traumatic brain injury and multiple fractures following a motor vehicle accident. He is
recovering in the intensive care unit. Your referral states PROM and positioning. On day 1 he is semi-alert and
drifts in and out while you are working with him. On day 2 you become concerned because you observe signs
suggestive of increasing intracranial pressure. You promptly report these symptoms to his physician. The signs that
would be cause for immediate action in this case would be:
A. decreasing consciousness with slowing of pulse and Cheyne-Stokes respirations.
B. decreasing function of cranial nerves IV, VI, and VII.
C. developing irritability with increasing symptoms of photophobia, disorientation and restlessness.
D. positive Kernigs sign with developing nuchal rigidity.

A119 C2 1
119. You have received a referral for a 42 year-old patient who has a neurapraxia involving the ulnar nerve secondary to
an elbow fracture. Based on your knowledge of this condition, you expect that:
A. regeneration is unlikely because surgical approximation of the nerve ends was not performed.
B. nerve dysfunction will be rapidly reversed, generally in 2-3 weeks.
C. regeneration is likely after 2-21/2 years.
D. regeneration is likely in 6-8 months.

A120 C7 3
120. After three weeks of teaching a patient how to ambulate with bilateral crutches and a touch down gait, you
determine the most appropriate kind of feedback to give to the patient is:
A. continuous feedback in which you provide ongoing verbal cuing during gait.
B. immediate feedback given after each practice trial.
C. intermittent feedback given at scheduled intervals, every other practice trial.
D. occasional feedback given when consistent errors appear.

A121 C8 2
121. You are a home health physical therapist. During one of your regularly scheduled visits with a 72 year-old male
patient, you find him to be confused with shortness of breath and significant generalized weakness. Assessing
these symptoms and given his history of hypertension and hyperlipidemia, you suspect:
A. he forgot to take his hypertension medication.
B. he may be experiencing unstable angina.
C. he may be presenting with early signs of myocardial infarction.
D. his mental changes are indicative of early Alzheimers disease.

A122 C1 2
122. Physical therapy intervention for a sixty-five year-old male patient with a recent diagnosis of supraspinatus
tendinitis with possible impingement syndrome of the right shoulder should emphasize:
A. joint mobilization, use of ice, and rotator cuff strengthening.
B. modalities to reduce inflammation, active assistive range of motion exercises using pulleys, and postural
realignment.
C. reducing stresses to abnormal tissues by placing the right upper extremity in a sling, use of ice, and rotator
cuff strengthening.
D. rest to reduce pain, iontophoresis, and strengthening of the rotator cuff muscles.

05 Sullivan A
20 Practice Questions

A123 C3 3
123. The recommended time duration for endotracheal suctioning is:
A. 1 to 5 seconds.
B. 10 to 15 seconds.
C. 15 to 20 seconds.
D. 5 to 10 seconds.

A124 C3 3
124. A 76 year-old frail older adult is confined to bed in a nursing facility. He has developed a small superficial wound
over the sacral area. Since only small amounts of necrotic tissue are present, the physician has decided to use
autolytic wound debridement. This is BEST achieved with:
A. forceful irrigations.
B. hydrotherapy.
C. occlusive dressings.
D. wet-to-dry dressings.

A125 C5 3
125. Three weeks ago, you instructed your patient in applying conventional (high rate) TENS to the low back to
modulate a chronic pain condition. The patient now states that the TENS unit is no longer effective in reducing the
pain in spite of increasing the intensity to maximum . You should now advise the patient to:
A. decrease the pulse duration.
B. increase the treatment frequency.
C. switch to low rate TENS.
D. switch to modulation mode TENS.

A126 C2 1
126. During initial standing a patient is pushing backward displacing the center-of-mass at or near the posterior limits
of stability. The most likely cause of this is contraction of the:
A. gastrocnemius-soleus.
B. hamstrings.
C. hip extensors.
D. tibialis anterior/peroneals.

A127 C7 2
127. A patient has been screened using a new test for the presence of a gene (ALG-2) linked to Alzheimers disease. His
physician reports he lacks the gene and should not be at increased risk to develop the disease. Some years later he
develops Alzheimers and a repeat test reveals the presence of the gene. The results of the initial test can be
interpreted as:
A. false negative.
B. false positive.
C. high degree of sensitivity.
D. high degree of specificity.

A128 C8 1
128. A 62 year-old lives at home with his wife and adult daughter. He has recently been diagnosed with multi-infarct
dementia and is recovering from a fractured hip following a fall injury. In your initial interview with his wife you
would expect to find:
A. agitation and sundowning.
B. history of steady progression of loss of judgment and poor safety awareness.
C. history of sudden onset of new cognitive problems and patchy distribution of deficits.
D. perseveration on a thought or activity.

05 Sullivan A
Practice Questions 21

A129 C1 2
129. A fourteen-year-old girl complains of subpatellar pain after participation in an aerobic exercise program for two
weeks. Your examination shows a large Q angle, pain with palpation at the inferior pole of the patella, and mild
swelling at both knees. Physical therapy intervention should promote:
A. hamstring strengthening.
B. lateral patellar tracking.
C. vastus lateralis strengthening.
D. vastus medialis muscle strengthening.

A130 C2 3
130. You receive a referral from an acute care physical therapist to treat a patient with right hemiparesis in the home.
The referral indicates that the patient demonstrates good recovery: both involved limbs are categorized as stage 4
(Brunnstrom recovery stages). He is ambulatory with an small-based quad cane. The activity that would be MOST
appropriate for a patient at this stage of recovery is:
A. sitting, marching in place (alternate hip flexion movements).
B. standing, picking the foot up behind and slowly lowering it.
C. standing, small range knee extension to gain quadriceps control.
D. supine, bending the hip and knee up to the chest with some hip abduction.

A131 C4 2
131. During a physical therapy session for low back pain, a 67 year-old patient tells you that she has had urinary
incontinence for the last year. It is particularly problematic when she has a cold and coughs a lot. She has not told
her doctor about this problem because she is too embarrassed. Your BEST course of action is to:
A. examine the patient and proceed with her back treatment.
B. examine the patient, document and discuss your findings with the doctor.
C. examine the patient, document the problems, then send her back to her doctor.
D. refer the patient back to her doctor.

A132 C2 1
132. During a sensory exam you determine that a patient complains of a dull, aching pain but is not able to discriminate
a stimulus as sharp or dull. Two-point discrimination is absent. Based on these findings, the pathway that is intact
is the:
A. anterior spinothalamic tract.
B. dorsal columns/neospinothalamic systems.
C. fasciculus gracilis/medial lemniscus.
D. lateral spinothalamic tract.

A133 C3 2
133. A patient is five days post-MI and is receiving cardiac rehabilitation. At this time a goal that would be
INAPPROPRIATE is to:
A. counteract deconditioning associated with bed rest.
B. educate the patient and family regarding risk factor reduction.
C. increase the patients maximal oxygen consumption by discharge.
D. initiate early return to independence in activities of daily living.

A134 C6 3
134. A 73 year-old man has bilateral short transfemoral (AK) amputations and will require a wheelchair for functional
mobility in the home and community. An appropriate prescription for his wheelchair includes:
A. increasing the seat depth by 2 inches to accommodate the length of the residual limbs.
B. lowering the seat height by 3 inches.
C. placement of the drive wheels 2 inches anterior to the vertical back supports.
D. placement of the drive wheels 2 inches posterior to the vertical back supports.

05 Sullivan A
22 Practice Questions

A135 C7 2
135. A multicenter study was done on the reliability of passive wrist flexion and extension goniometric measurements
using volar/dorsal alignment, ulnar alignment and radial alignment. Significant differences were revealed between
the three techniques. An appropriate level for determining significant difference is a P value of:
A. P=0.015
B. P=0.05
C. P=0.1
D. P=0.5

A136 C1 3
136. In treating a patient with a diagnosis of right shoulder impingement syndrome, the therapist should not allow the
patient to perform assisted repetitive overhead exercises without FIRST:
A. controlling all pain.
B. having complete AROM at the shoulder.
C. instruction in proper postural alignment.
D. stretching the shoulder girdle muscles.

A137 C8 3
137. An older person with a diagnosis of degenerative joint disease should understand his disease, its progression, and
its management. You will know your patient education has been effective if he can tell you:
A. aerobic conditioning is not appropriate when pain is present and medications must be used.
B. joint protection strategies are important but cannot reduce the expected loss of function.
C. loss of ROM and immobility are expected and irreversible.
D. pain and stiffness are worse in the early morning and should decrease with moderate activity.

A138 C2 1
138. A 34 year-old patient is recently admitted to your facility with a diagnosis of Guillain-Barr syndrome. In your
initial examination, you expect to find:
A. asymmetrical weakness with hyperreflexia, bulbar palsy.
B. glossopharyngeal and vagal paralysis with hyperactive jaw and snout reflexes.
C. sensory loss (stocking and glove distribution) with minor loss of motor function.
D. symmetrical distribution of weakness, ascending with possible involvement of lower cranial nerves.

A139 C5 3
139. A 31 year-old patient presents with patellar tendinitis as a result of a mountain climbing accident three weeks ago.
The patient complains of pain on resisted knee extension, stair climbing, and sit-to-standing movements. You
choose to apply iontophoresis using a pain medication with a positive charge. The correct current type, polarity
and active electrode placement is:
A. biphasic current with the cathode placed proximal on the tendon.
B. low volt continuous current with the anode placed distal on the tendon.
C. monophasic current with the anode placed on the tendon.
D. monophasic current with the cathode placed on the tendon.

A140 C6 3
140. A 52 year-old patient sustained a T10 spinal cord injury four years ago. During initial examination you observe
redness over the ischial seat that persists for 10 minutes when not sitting. The BEST intervention in this case
would be to:
A. change to a low density wheelchair cushion.
B. have the patient do sitting push-ups at least every 10 minutes.
C. increase the arm rest height.
D. order a tilt-in-space wheelchair.

05 Sullivan A
Practice Questions 23

A141 C1 1
141. A 16 year-old adolescent female was sent to physical therapy with a diagnosis of anterior knee pain. Positive
findings include pes planus, lateral tibial torsion and genu valgum. The position the femur will be in is excessive:
A. abduction.
B. lateral rotation.
C. medial rotation.
D. retroversion.

A142 C2 3
142. A 24 year-old patient in a coma recovering from traumatic brain injury is receiving PROM exercises. The family is
confused because the occupational therapist has told them to do range of motion exercises in a different way than
the physical therapist had instructed. As the physical therapist it would be BEST to deal with this situation by
telling the family:
A. the PT and OT supervisors will review and correct any discrepancies in the exercise program.
B. there is no need for them to worry and note the problem in the patients record.
C. to exercise the way you instructed until any differences can be worked out with the occupational therapist.
D. you will meet with the occupational therapist to discuss the exercise approaches and you will let them know
the outcome of the meeting as soon as possible.

A143 C7 3
143. You are working with a 10 year-old girl with cerebral palsy. Part of the exercises in her plan of care involve using
the Swiss ball. The choice of educational media that is BEST to use when instructing her in use of this device is:
A. a oral presentation that uses transparencies of Swiss ball positions.
B. a slide presentation of exercises using the Swiss ball.
C. a videotape of another child with cerebral palsy on a Swiss ball.
D. printed handouts with stick figure drawings and instructions.

A144 C3 3
144. As a physical therapist you are the health professional in charge during a high school football game. During the
game, a player is tackled violently by two opponents. You determine that the player is unresponsive. Your
immediate course of action should be to:
A. ask for help to log roll the player on his back while stabilizing his neck.
B. open the airway by using the chin-lift method.
C. stabilize the neck and flip back the helmet face mask.
D. summon emergency medical services.

A145 C8 2
145. During a home visit, the mother of an 18 month-old child with developmental delay and an atrio-ventricular shunt
for hydrocephalus tells you that her daughter vomited several times, was irritable and is now lethargic. Your BEST
course of action is to:
A. call the girls pediatrician immediately.
B. give the child a cold bath to try and rouse her.
C. give the child clear liquids since she vomited.
D. place the child in a sidelying position and monitor vital signs.

A146 C4 2
146. A 24 year-old woman recently delivered twins on the obstetrical service of your hospital. After delivery she has
developed a 4 centimeter diastasis recti abdominis. The BEST initial intervention for this problem is to teach:
A. gentle stretching of hamstrings and hip flexors.
B. pelvic floor exercises and sit-ups.
C. pelvic tilts and bilateral straight leg raising.
D. protection and splinting of the abdominal musculature.

05 Sullivan A
24 Practice Questions

A147 C3 3
147. The optimal position for ventilation of a patient with a C5 complete spinal cord injury is:
A. semi Fowlers.
B. sidelying, head of bed elevated 45 degrees.
C. sidelying, head of bed flat.
D. supine, head of bed flat.

A148 C7 2
148. A 16 year-old patient with osteosarcoma is being seen in physical therapy for crutch training. Her parents have
decided not to tell her about her diagnosis. She is quite perceptive and asks you directly if she has cancer and
about her future. Your BEST course of action is to:
A. change the subject and discuss the plans for that days treatment.
B. discuss her condition gently indicating her parents fears about not telling her the diagnosis.
C. schedule a conference with the doctor and family about her condition and your discussions with the patient.
D. tell the patient that you dont know the specifics of her condition or prognosis, and she should speak with her
doctor.

A149 C2 2
149. Your patient has had amyotrophic lateral sclerosis for the past two years with mild functional deficits. He is still
ambulatory with bilateral canes but is limited in his endurance. An important goal for his physical therapy plan of
care should be to prevent:
A. further gait deterioration as a result of ataxia.
B. myalgia.
C. overwork damage in weakened, denervated muscle.
D. radicular pain and paresthesias.

A150 C6 3
150. A 28 year-old patient has extensive full thickness burns to the dorsum of the hand and forearm. He is to be fitted
with a resting splint to support his wrists and hands in a functional position. An appropriately constructed splint
positions the wrist and hand in:
A. neutral wrist position with IP extension and thumb flexion.
B. neutral wrist position with slight finger flexion and thumb flexion.
C. slight wrist extension with fingers supported and thumb in partial opposition and abduction.
D. slight wrist flexion with IP extension and thumb opposition.

A151 C3 1
151. Your patient is 82 years-old with a long history of congestive heart failure. You are alert to the signs of left-sided
heart failure associated with activity. The MOST important indicators include:
A. bilateral ankle swelling within 2-3 hours post exercise.
B. complaints of fatigue with increasing dyspnea and cough.
C. nausea and anorexia.
D. sudden weight gain within 24 hours following exercise.

A152 C3 2
152. A patient using an incentive spirometer complains of feeling lightheaded. Your instructions to the patient should be
to:
A. lie down while using the spirometer.
B. take a deeper breath on the following attempt.
C. take a rest period and only use the device 10 times per hour.
D. try to use the spirometer more frequently to get used to it.

05 Sullivan A
Practice Questions 25

A153 C8 3
153. An eighteen month-old child with Down Syndrome and moderate developmental delay is being treated at an Early
Intervention Program. The schedule that would best facilitate motor learning of this child is physical therapy
intervention given:
A. 30 minutes each day.
B. once a week for two hours.
C. one hour each week with a portion of that time used to teach the caretaker a home program to be done 3 times
a week.
D. three times a week for fifteen minutes.

A154 C1 1
154. A patient has been referred to you for acute shoulder pain after shoveling snow in a driveway for two hours.
Positive findings include pain and weakness with flexion of an extended upper extremity as well as scapular
winging with greater than 90 degrees of abduction. The patients problem is MOST LIKELY the result of:
A. compression of the long thoracic nerve.
B. compression of the suprascapular nerve.
C. subdeltoid bursitis.
D. supraspinatus tendinitis.

A155 C2 1
155. A patient with multiple sclerosis demonstrates strong bilateral lower extremity extensor spasticity in the typical
distribution of antigravity muscles. You would expect this patient to demonstrate:
A. sacral sitting.
B. sitting with both legs abducted and externally rotated.
C. sitting with the pelvis tilted, weight bearing on ischial tuberosities.
D. skin breakdown on the ischial tuberosities and lateral malleoli.

A156 C4 1
156. A 58 year-old woman with osteopenia has been on Premarin for the past 4 years. Based on your knowledge of
estrogen replacement therapy, you recognize this patient is more susceptible to:
A. fractures.
B. osteoporosis.
C. peripheral edema.
D. vasomotor symptoms (hot flashes).

A157 C4 2
157. A 24 year-old pregnant woman who is 12 weeks pregnant asks you if it is safe to continue with her aerobic
exercise. Currently she jogs 3 miles, 3 times a week. Your BEST answer is:
A. continue jogging only until the 5th month of pregnancy.
B. during the last trimester do not jog but switch to exercising in the supine position only.
C. jogging is safe but you might want to switch to swimming during later months.
D. jogging is safe but your target HR should not exceed 140 beats/min.

A158 C6 3
158. Your patient returns to P.T. after his first exercise session complaining of muscle soreness that developed later in
the evening after his first session and continued into the next day. He is unsure he wants to continue with exercise.
You can minimize the possibility of this happening again by using:
A. concentric exercises, 3 sets of 10, at 80% of maximal intensity.
B. concentric exercises, 3 sets of 10, with gradually increasing intensity.
C. eccentric exercises, 1 set of 10, lifting body weight (sit-to-stand).
D. eccentric exercises, 3 sets of 10, with gradually increasing intensity.

05 Sullivan A
26 Practice Questions

A159 C1 1
159. Upon examining a patient with vague hip pain which radiates to the lateral knee, you have found a negative
FABERE test, negative grind test, and a positive Noble compression test. The dysfunction is most likely due to:
A. a possible fracture of the femoral neck.
B. an iliotibial band friction disorder.
C. degenerative joint disease of the hip.
D. sacroiliac joint dysfunction.

A160 C2 1
160. A 22 year-old male suffered carbon monoxide poisoning from a work-related factory accident. He is left with
permanent damage to his nervous system, affecting the basal ganglia. You logically expect his symptoms to
include:
A. impaired sensory organization of balance.
B. motor paralysis.
C. muscular spasms and hyperreflexia.
D. problems with motor planning and scaling of movements.

A161 C2 1
161. A patient presents with problems with swallowing. When you test for phonation by having the patient say AH
with his mouth open, you notice there is deviation of the uvula to one side. You then test for function of the gag
reflex and notice loss of response to stimulation. These findings suggest involvement of the:
A. facial nerve.
B. hypoglossal nerve.
C. trigeminal nerve.
D. vagus nerve.

A162 C3 1
162. A patient with COPD is sitting in a bedside chair. The apices of the lungs in this position compared with other
areas of the lungs in this position would demonstrate:
A. increased perfusion.
B. increased volume of air at resting end expiratory pressure (REEP).
C. the highest changes in ventilation during the respiratory cycle.
D. the lowest oxygenation and highest CO2 in blood exiting this zone.

A163 C6 1
163. Checkout for a lower limb orthosis includes inspection of the alignment of anatomic and orthotic joints. During a
sagittal plane check-out you determine that the orthotic hip joint is malaligned. The correct position is:
A. 3 inches below the anterior superior iliac spine.
B. just anterior and superior to the greater trochanter.
C. just posterior and inferior to the greater trochanter.
D. lateral to the greater trochanter.

A164 C7 3
164. A physical therapist assistant you supervise treated a patient in the home care setting. The patient is status post
CVA. Part of the plan of care includes progressive gait training on level surfaces. The patient falls and sustains a
fractured hip during a visit done by the PTA. The fall occurred when the PTA took the patient on the stairs for the
first time. The responsible party in this case is:
A. both the PT and the PTA because the PT gave inadequate supervision, and the PTA used poor judgment.
B. neither the PT nor the PTA because patients who have sustained a CVA are always at high risk for falling, and
thus it is a regrettable occurrence only.
C. the PT who is negligent for failing to provide adequate supervision of the PTA.
D. the PTA who is completely liable because the plan of care was altered without communicating with the
supervising PT.

05 Sullivan A
Practice Questions 27

A165 C1 3
165. As the result of blunt trauma to the quadriceps femoris muscle, a patient experiences loss of knee function. Early
PT interventions should stress:
A. aggressive open-chain strengthening of the quadriceps femoris to regain normal lower extremity strength.
B. aggressive soft tissue stretching to remove blood which has accumulated in soft tissues.
C. gentle AROM exercises in weight bearing.
D. gentle PROM exercises in nonweightbearing to regain normal knee motion.

A166 C8 3
166. An important adjunct to physical therapy management of a child with moderate spastic hemiplegia would be use
of:
A. a KAFO on the affected side.
B. a posterior walker.
C. a tone inhibiting ankle-foot orthosis (AFO).
D. an anterior rollator walker.

A167 C2 1
167. While evaluating the gait of a patient with right hemiplegia, you note foot drop during midswing on the right. The
MOST LIKELY cause of this deviation is:
A. decreased proprioception.
B. excessive extensor synergy.
C. excessive flexor synergy.
D. inadequate contraction of the ankle dorsiflexors.

A168 C7 3
168. A patients daughter wants to look at her fathers medical record. He has recently been admitted for an insidious
onset of low back pain. You, as the physical therapist, should:
A. give her the chart and let her read it.
B. tell her she cannot see the chart because she could misinterpret the information.
C. tell her that she must have the permission of her father before she can look at the chart.
D. tell her to ask the physician for permission

A169 C5 2
169. A 48 year-old female has had a total knee replacement. Following surgery, you place her on a regimen of
continuous passive motion. The primary purpose for applying CPM for the first few hours is to:
A. decrease edema.
B. decrease pain.
C. increase range of motion.
D. increase tissue tensile strength.

A170 C5 2
170. A 37 year-old male developed acute bicipital tendinitis two days after water skiing. He is unable to work as a result
of the pain he rates as 8/10. He is referred to physical therapy for iontophoresis treatment to help relieve the pain.
For the first two weeks this patient should optimally be seen:
A. five times per week
B. once a week.
C. three times per week.
D. two times per week

A171 C6 1
171. You observe genu recurvatum as a patient with hemiplegia ambulates using his posterior leaf spring (PLS)
orthosis. A factor that is NOT a possible cause of this gait deviation is:
A. extensor spasticity.
B. hip flexion contracture.
C. pes equinus.
D. quadriceps weakness.

05 Sullivan A
28 Practice Questions

A172 C3 2
172. During a home visit you are providing postural drainage in the Trendelenburg position to a 15 year-old male with
cystic fibrosis. The patient suddenly complains of right-sided chest pain and shortness of breath. On auscultation,
there are no breath sounds on the right. The physical therapist should:
A. call emergency medical technicians as it may be a pneumothorax.
B. continue treating as it is possibly a mucous plug.
C. place the right lung in a gravity dependent position to improve perfusion.
D. reposition patient in the head of bed flat position as Trendelenburg is causing shortness of breath.

A173 C2 3
173. A 65 year-old patient with multiple sclerosis is being treated at home. The patient is bedridden for most of the day
with only short periods up in a bedside chair. Medicare is funding the patients home care program which has as its
primary goals maintaining PROM and positioning to prevent deformity. Your role as the physical therapist is to
provide:
A. a limited cardiovascular conditioning (sitting) program aimed at improving respiratory capacity.
B. a restorative exercise program aimed at improving upright sitting control and improved functional
independence.
C. PROM exercises 2 times a day with additional family instruction to ensure weekend coverage.
D. supervision of home health aides for completion of a daily home exercise program.

A174 C2 3
174. Your patient is a 72 year-old man with an 8 year history of Parkinsons disease. He demonstrates significant
rigidity, decreased PROM in both upper extremities in the typical distribution, and frequent episodes of akinesia.
The exercise that BEST deals with these problems would be:
A. modified plantigrade, isometric holding, stressing upper extremity shoulder flexion.
B. PNF bilateral symmetrical upper extremity D2 flexion patterns, rhythmic initiation.
C. quadruped position, upper extremity PNF D2 flexion and extension.
D. resistance training, free weights for shoulder flexors at 80% of 1 repetition max.

A175 C7 3
175. A 72 year-old patient with diabetes is recovering from recent surgery to graft a large decubitus ulcer over the heel
of her left foot. You are concerned that her loss of range at the ankle (-5 to neutral) will limit her ambulation and
independent status. One afternoon you are very busy and request that one of the physical therapy aides do her
range of motion exercises. The aide is new to your department but tells you she is willing to take this challenge on
if you show her how to do it. Your BEST course of action is to:
A. modify the treatment session to eliminate the ROM exercises and have the aide walk the patient in the parallel
bars.
B. perform the ROM exercises yourself.
C. reschedule the patient for tomorrow when you have more time.
D. take 5 minutes to instruct the aide in ROM exercises, then have her do them.

A176 C8 2
176. The MOST appropriate type of adaptive equipment for a teenager with severe athetosis and severe extensor
spasms of the lower extremities would be:
A. posterior rollator walker.
B. scooter board.
C. supine stander with head support.
D. wheelchair with head and trunk support with abductor wedge.

A177 C1 2
177. The symptoms of ankylosing spondylitis in its early stages can best be managed in physical therapy by:
A. back extension, costal expansion exercises, and maintenance of proper posture to prevent deformity.
B. cardiovascular conditioning, weight bearing exercises, and joint protection education.
C. pain management, abdominal strengthening, and breathing exercises.
D. strengthening of anterior chest muscles, costal expansion, and stretching of scapular stabilizers.

05 Sullivan A
Practice Questions 29

A178 C2 1
178. A 72 year-old male was referred for rehabilitation following a middle cerebral artery stroke. Based on this
diagnosis you suspect he will present with:
A. aphasia since his right hemisphere is involved and he is right hand dominant.
B. contralateral hemiparesis and sensory deficits with greater involvement in the arm than the leg.
C. contralateral hemiparesis and sensory deficits with greater involvement of leg than the arm.
D. homonymous hemianopsia, contralateral hemiplegia with thalamic sensory syndrome.

A179 C3 1
179. The cardiac rehabilitation team is conducting education classes for a group of patients. The focus is on risk factor
reduction and successful life style modification. A participant asks you which is the bad cholesterol that increases
atherosclerosis and risk of heart disease. Your answer is:
A. all cholesterol is bad and total levels should be below 200 mg/dl.
B. cholesterol is not the important factor; the triglycerides should be kept low.
C. elevated levels of high-density lipoprotein or HDL.
D. elevated levels of low-density lipoprotein or LDL.

A180 C7 3
180. An attractive physical therapist is treating a young and handsome football player with an ACL sprain. She is very
fond of this patient and enjoys treating him. After a few visits, the football player asks her out to dinner. The
physical therapists response should be to:
A. thank him very much, and accept his offer for dinner.
B. thank him very much, and invite him for dinner at her apartment with other guests.
C. thank him very much, but refuse his invitation while he is receiving treatment.
D. transfer the patient care to one of her colleagues and then go out to dinner with him.

A181 C1 2
181. After running one mile, an athlete complains of deep cramping at the anterior aspects of the legs which does not
ease and prevents continued running. Management of this problem should include:
A. orthotic fabrication to enable continued running on all surfaces
B. referral to a physician to evaluate anterior compartment pressures during activity.
C. referral to a physician to rule out spinal stenosis.
D. stretching of the tibialis anterior muscles to help resolve shin splints.

A182 C2 2
182. A 62 year-old woman developed polio at the age of 6 with significant lower extremity paralysis. She wore bilateral
long leg braces for a period of 2 years. She then recovered enough to stop using her braces but still required
bilateral Lofstrand crutches, then bilateral canes to ambulate. Recently she has been complaining of new
difficulties (she has had to start using her crutches again). You suspect post-polio syndrome. The BEST
goal/intervention for this patient based on her current findings is to:
A. prescribe an aquatic therapy program consisting of daily 1 hour aerobics.
B. put her on a lower extremity resistance training program utilizing 80% one repetition max.
C. teach her activity pacing and energy conservation techniques.
D. utilize a moderate conditioning program consisting of cycle ergometry 3 times a week of 60 minutes at 60%
maximal heart rate.

A183 C1 1
183. A patient diagnosed with lumbar spondylosis without discal herniation or bulging has a left L5 neural
compression. The most likely structure compressing the nerve root is:
A. anterior longitudinal ligament.
B. ligamentum flavum.
C. posterior longitudinal ligament.
D. supraspinous ligament.

05 Sullivan A
30 Practice Questions

A184 C3 2
184. During an exercise tolerance test (ETT) a patient demonstrates poor reaction to increasing exercise intensity.
According to the American College of Sports Medicine, an absolute indication for terminating this test is:
A. 1.5 mm of downsloping ST-segment depression.
B. fatigue and shortness of breath.
C. onset of moderate to severe angina.
D. supraventricular tachycardia.

A185 C2 3
185. A patient with T10 paraplegia is receiving daily ROM exercises. On this day you notice swelling, pain, local
warmth and erythema in the thigh near the hip joint with some limitation in motion. Your BEST course of action
is:
A. apply ice before his ROM exercises to ease his pain and discomfort and inflammation.
B. document the findings in his record and double his daily sessions of ROM exercises.
C. instruct his aides to be more vigorous in positioning and ROM exercises.
D. notify the physician immediately, you suspect heterotopic ossifications may be developing.

A186 C6 1
186. A patient with a T4 spinal cord injury is being measured for a wheelchair. In determining the correct seat height
you can use as a measure:
A. clearance between the floor and the foot plate of at least 2 inches.
B. clearance between the floor and the foot plate of at least 4 inches.
C. the distance from the bottom of the shoe to just under the thigh at the popliteal fossa.
D. the patients leg length measurement plus four inches.

A187 C8 2
187. A 6 year-old boy has a diagnosis of Duchenne muscular dystrophy and is still ambulatory. The MOST appropriate
activity to include in his plan of care would be:
A. circuit training program.
B. progressive resistance strength training.
C. recreational activities such as swimming or biking.
D. wheelchair sports.

A188 C7 3
188. A 72-year old patient is receiving outpatient physical therapy at your private clinic. The clinic is an approved
Medicare outpatient provider. Your patient is concerned that she will not be able to pay for her continuing care and
worries that her Medicare benefits will run out soon. You tell her:
A. coverage is limited only for hospital-based outpatient PT services.
B. currently there is no limit to her Medicare coverage for outpatient PT services.
C. there is currently a limit of $1000.00 for coverage of outpatient PT services.
D. there is currently a limit of $1590.00 for coverage of outpatient PT services.

A189 C2 3
189. Your patient is a 42 year-old woman who suffered a stroke and demonstrates a locked-in state characterized by
spastic quadriplegia and bulbar palsy. To facilitate communication with this patient you should instruct the family
to:
A. give her a chance to mouth her responses even though she cant vocalize well.
B. look closely at her facial expression to detect signs of what she is trying to communicate.
C. use a communication board with minimal movements of her hand.
D. use an alternate eyelid taping schedule which will allow her to use eye movements to communicate.

A190 C6 2
190. After performing an ergonomic examination of a worker and workstation the most appropriate recommendation
for achieving ideal wrist and elbow positioning would be to:
A. add armrests.
B. elevate the keyboard to increase wrist flexion.
C. lower the keyboard to increase wrist extension.
D. maintain the keyboard in a position allowing a neutral wrist position.

05 Sullivan A
Practice Questions 31

A191 C3 1
191. A 43 year-old patient with a post myocardial infarction is on digitalis to improve cardiac contractility. He is a new
participant in your Phase 2 outpatient cardiac rehabilitation program. He is being continuously monitored by ECG
via radio telemetry. On his ECG, the medication induced changes that could be expected are:
A. decreased heart rate with prolonged QRS and QT intervals.
B. depressed ST segment with a flat T wave and shortened QT interval.
C. elevated ST segment with T wave inversion.
D. widened QRS complex with a flattened P wave.

A192 C3 2
192. Your patient has lymphatic disease of the right arm secondary to radical mastectomy and radiation. The resulting
edema can BEST be managed in physical therapy by:
A. AROM and extremity positioning.
B. intermittent pneumatic compression, extremity elevation, and massage.
C. isometric resistive exercises and extremity positioning in elevation.
D. PROM and extremity elevation.

A193 C7 2
193. You have been treating a patient for chronic subluxation of the patella in the outpatient clinic. He is now scheduled
for a lateral release and is worried about any complications of the surgical procedure. He asks you to describe any
potential complications. Your BEST response is to:
A. do a search on the internet and get back to the patient with the desired information.
B. explain how patients you have treated responded to the surgery.
C. refer the patient to a physical therapy colleague who specializes in knee problems.
D. suggest that the patient speak with his surgeon.

A194 C1 3
194. A 46 year-old female was referred for physical therapy following a right breast lumpectomy with axillary lymph
node dissection. Scapular control is poor when upper extremity flexion or abduction is attempted. PT intervention
should focus on:
A. active assistive pulley exercises to assist rotator cuff muscles as a result of damage to the suprascapular nerve.
B. gravity assisted right upper extremity exercises to promote scapular control following damage to the long
thoracic nerve.
C. rhomboid strengthening as a result of disuse of the scapular stabilizers.
D. strengthening of the right deltoids to help stabilize the shoulder to compensate for damage to the dorsal
scapular nerve.

A195 C3 3
195. A post surgical patient is receiving a regimen of postural drainage three times a day. You could reduce the
frequency of treatment if the:
A. amount of productive secretions decreases.
B. consistency of the sputum changes.
C. patient becomes febrile.
D. patient experiences decreased postoperative pain.

A196 C2 1
196. The distinguishing feature of Mnires disease which sets it apart from other peripheral conditions affecting the
vestibular system is:
A. chronic, episodic bouts of dysfunction.
B. postural instability.
C. significant dysfunction with severe attacks of vertigo and nausea.
D. time-limited, usually lasting only 24-48 hours.

05 Sullivan A
32 Practice Questions

A197 C7 3
197. A physical therapist arriving at work one hour before she was due to start work, began moving treatment tables
and rearranging the physical therapy clinic. This operation could have been done during regular hours. The
therapist sustained a low back injury as a result of moving the equipment. Payment for the therapists care relating
to this incident would be covered primarily by:
A. employees health insurance.
B. the hospitals insurance company.
C. the therapists own resources.
D. Workers Compensation.

A198 C1 2
198. Patients diagnosed with Pagets disease typically have similar symptomatology to spinal stenosis. The most
important aspect of physical therapy intervention emphasizes:
A. lumbar extension exercises.
B. modalities to decrease pain.
C. postural reeducation to prevent positions that increase symptoms.
D. strengthening exercises for the abdominals and back muscles.

A199 C6 1
199. A patient with spastic left hemiplegia experiences severe genu recurvatum during stance phase. If the patient is
using an ankle-foot orthosis, the cause of the problem might be attributed to:
A. the anterior stop setting the foot in too much dorsiflexion.
B. the anterior stop setting the foot in too much plantar flexion.
C. the posterior stop setting the foot in too much dorsiflexion.
D. the posterior stop setting the foot in too much plantarflexion.

A200 C3 3
200. A patient has arterial peripheral vascular disease with symptoms of intermittent claudication in both lower
extremities. Which of the following is NOT an appropriate guideline for exercise training?
A. daily walking through pain in order to attain predetermined distances.
B. daily walking, 2 to 3 times a day.
C. interval training protocol with frequent rests.
D. stopping exercise and resting once the pain threshold is reached

05 Sullivan A
Practice Questions 33

1. glenohumeral mobilization and strengthening of scapular stabilizers to regain normal scapulohumeral movement.
Compensation of glenohumeral restrictions if often exhibited as excessive scapular movement. Therefore,
mobilization of the glenohumeral joint and strengthening of scapular stabilizers is needed to regain normal
scapulohumeral motion.

2. unilateral hearing loss. Presbycusis is a sensorineural hearing loss associated with aging. It is characterized by
bilateral hearing loss with deficits noted in auditory discrimination and comprehension.

3. increase the warm-up and cool-down periods to equal the total aerobic interval in time. Clinical decisions should
focus on reducing the environmental costs of exercising (change the time of the day of the exercise class to reduce
the heat stress) or reducing the overall metabolic costs of the activity (decrease the pace of the exercise, add more
rest periods). Altering the warm-up and cool-down periods does not lower the overall cost of the aerobic exercise
period.
4. anterior tibia, tibial crest, and fibular head. In the PTB socket, reliefs are provided for pressure sensitive areas: the
anterior tibia and tibial crest, fibular head, and peroneal nerve. All the other choices are considered pressure
tolerant areas.

5. take time now to allow the patient to express her fears and frustrations. It is important to be supportive of a patient
who is experiencing losses and resentment. Allow the patient to fully verbalize her feelings and frustrations.

6. 1 MHz continuous at 1.0 W/cm2. 1 MHz of continuous ultrasound provides deep heating to a depth of 3-5 cm. At
this frequency, attenuation (absorption) is les in superficial tissues. His allows more energy to be absorbed, thus
more heat production in deeper tissue layers. Continuous US is applied to achieve thermal effects (i.e., chronic
pain) and pulsed US is used when nonthermal effects are desired (i.e., acute soft tissue injuries).

7. some recovery of function since damage is to peripheral nerve roots. A spinal cord lesion below L1 is a cauda
equina lesion (injury to the peripheral roots and nerves). Since some regeneration is possible, some recovery in
function can be expected. A spastic or reflex bladder is associated with upper motor neuron injury. Other choices
describe the deficits associated with anterior cord syndrome or central cord syndrome.

8. be designated as a more advanced task and more appropriately delegated to another physical therapist. Physical
therapy students should not perform advanced tasks for the first time without any instruction or direct supervision.
This might be unsafe for the patient. The task of orthotic checkout is most appropriately performed by another
physical therapist.

9. instruction to eat a soft food diet and phonophoresis. Phonophoresis and education regarding consumption of only
soft food with help resolve the acute inflammatory process in the temporomandibular joint. Application of an
intraoral appliance occurs only when the acute inflammation is not resolved or bruxism continues. Joint
mobilization should not be attempted with an acute inflammation.

10. posterior right C7 articular pillar. The most effective hand placement for mobilization into great left rotation is at
the posterior aspect of the right C7 articular pillar because it rotates the C7 vertebra to the left.

11. transfer objects from one hand to another. Transferring objects from one hand to another is a task
developmentally appropriate for an 8-9 month-old. Using a fine pincer grasp and building a tower of 4 blocks are
skills which develop later. Holding a cup by the handle while drinking usually occurs by 12 months of age.

12. document your observations and refer him back to his physician for evaluation of possible dopamine toxicity.
Dyskinesias (involuntary movements) are a common side effect of dopamine toxicity as are gastrointestinal
disturbances (nausea, vomiting) and mental disturbances (restlessness, general overactivity, anxiety or depression).
While the symptoms described in the other choices may also occur with Parkinsons disease, they do not
adequately explain the presence of adventitious or involuntary movements.

13. explain how patients typically respond to the surgery and outline the progression of exercises. Assess the needs of
the patient and provide appropriate information based on the expected rehabilitation process. Do not pass the
buck unless the information is outside of the scope of your expertise.

05 Sullivan A
34 Practice Questions

14. patient reaches age-predicted maximal heart rate. A maximum exercise tolerance test is a sign or symptom
limited test. Achieving age adjusted predicted maximum heart rate is not a sign or symptom and therefore does not
stop the test.

15. swing-to. A swing-to gait pattern is indicated to individuals with limited use of both lower extremities and trunk
instability. It is slower and more stable than a swing-through gait pattern (a gait pattern this patient can be
progressed to after his initial training). This patient is unable to perform a reciprocal gait.

16. request her primary physician to refer her for psychological evaluation. The patient is experiencing grief over her
loss. Significant persistent symptoms are an indication for a referral to a qualified professional (psychologist) to
help her deal with her loss.

17. friction. Friction massage is effective when applied across (perpendicular) muscle fibers to stretch scars and
loosen adhesions in the tissue due to the inflammatory process. Stroking is a relaxation technique and usually
initiates and ends massage treatment. Some passive muscle stretching is performed with deep stroking. Kneading
aids in loosening adhesions and increasing venous return. Tapotement is used when nerve stimulation or lung
decongestion is the desired treatment effect.

18. vigorous range of motion of the lower extremities. Vigorous range of motion is contraindicated due to osteoporosis
and risk of fracture. All of the other choices are appropriate to promote functional independence.

19. SI joint. Pain at end range of flexion, abduction, and external/lateral rotation (FABERE Test) is diagnostic for SI
joint dysfunction because it both gaps and compresses the joint. Pain at the midrange into hip flexion, abduction
and external/lateral rotation suggests hip joint pathology.

20. chi square test. Chi square is nonparametric statistical test used to compare data in the form of frequency counts
(in this example frequencies of carpal tunnel syndrome) occurring in two different groups of workers.

21. median. The mean is a measure of central tendency that is calculated by adding up all the scores and dividing the
total by the number of scores. The median is a middle score while the mode is the most frequently occurring score.
The most accurate measure of performance is skewed distribution with extreme scores in the median.

22. denervation atrophy has occurred. Spontaneous fibrillation potentials present on EMG are evidence of
denervation atrophy. Polyphasic motor units of low amplitude and short duration are evidence of reinnervation.

23. one-arm drive chair. A one-arm drive wheelchair has both drive mechanisms located on one wheel. The patient
can propel the wheelchair by using one hand(in this case his sound left hand). It is contraindicated in patients with
cognitive or perceptual deficits. A hemiplegic chair (low seat height) allows use of both the sound hand and leg to
propel the chair. The electric wheelchair with joystick might also work but is significantly more expensive, less
transportable, and would require increased maintenance. An elevating legrest is needed to complete the wheelchair
prescription.

24. functional residual capacity. Muscular dystrophy will alter the muscles ability to pull in air and blow out air;
therefore, vital capacity, total lung capacity, and forced expiratory volume in one second will be decreased. Since
muscular dystrophy does not change the lung parenchyma, REEP will occur at same point of equilibrium between
lung recoil and thoracic outward pull. Therefore, functional residual volume will not change.

25. swing phase. Foot drop is a swing phase deficit. Stimulation of the dorsiflexor muscles during the swing phase
places the foot in a more neutral position and prevents the toes from contacting the ground and interfering with the
gait pattern.

26. refuse to treat that patient. Blood and Body Fluid Precautionary Guidelines from the Centers of Disease Control
(CDC) state that a healthcare worker with exudative lesions or weeping dermatitis should refrain from all direct
patient care and from handling patient-care equipment until the condition resolves.

27. tight iliopsoas muscle. Tight iliopsoas muscles are common after THR surgery because the patient typically
spends more time in a sitting position. The iliopsoas attaches to the transverse processes of the lumbar spine and
the lesser trochanter of the femur. Therefore, when the iliopsoas is tight, the lumbar spine will extend if the femur

05 Sullivan A
Practice Questions 35

is fixed. In supine, the hamstrings and piriformis muscles are on slack and would not cause alteration in spinal
position.

28. environmental changes, a bedside commode, and referral for home health services. Clinical decision making in
this case should focus on the patients ability to manage in the home. Environmental modifications (the addition of
a commode) and assistance of a home care aide should allow her to safely return home. The referral for home
physical therapy should focus on improving her endurance to regain independence in the home. Treatment at home
is the most cost effective in this case.

29. make the patient aware of his deficit and teach him to turn his head to the affected left side. A patient with
homonymous hemianopsia needs to be made aware of his deficit and instructed to turn his head to the affected left
side (a compensatory training strategy). Initial strategies include placing items or doorway on his right (unaffected
side) so he can successfully interact with his environment. Later, as he demonstrates the ability to compensate,
items can be moved to midline and finally to his affected left side.

30. reduce the exercise intensity and provide relaxation strategies. Common problems in multiple sclerosis include
fatigue and heat tolerance. The agitation and irritation are most likely a result of frustration and fatigue. Exercise
intensity should be reduced and relaxation strategies utilized. A cool environment is important to reduce fatigue
but would not address the problems of agitation and irritation. A warm pool is contraindicated.

31. Medicare. Custodial care of a medically stable individual (over 65) who needs assistance with basic activities of
daily living such as eating, dressing, toileting and bathing in the home is covered by Medicare only when skilled
care (the services of a professional nurse or therapist under a physician-authorized plan of home care) is required.

32. instruct in proper dressing changes and wound care. Proper dressing changes and wound care to prevent
secondary wound infection are the hallmarks of conservative management of venous ulcers. Pliable,
nonstretchable dressing wraps (e.g., Unna boot) or custom-fitted support stockings can be used to assist in venous
circulation while elastic wraps do little to assist circulation. Prolonged hydrotherapy is contraindicated for venous
ulcers.

33. spasticity or contracture of the plantarflexors. Forward advancement of the tibia from midstance to heel-off is
controlled by eccentric contraction of the plantarflexors; from heel-off to toe-off the plantarflexors contract
concentrically. Either spasticity or contracture of the plantarflexors would limit this forward progression. Patients
compensate by going right into swing, typically with circumducted gait, or with increased hip and knee flexion
since there is no push off.

34. thumb abduction. Thumb abductors are innervated by the median nerve, primarily by the C6 nerve root. The
anterior divisions contribute to nerves that primarily serve flexors, and in this case, the thumb.

35. be calm and supportive, using only one or two level commands. An agitated patient with dementia does not
process information easily. A calm and supportive approach with low level commands (one or two actions)
provides the best approach to take with this patient.

36. postural hypotension. Fall risk is significantly increased with certain medications. Tricyclic antidepressants (e.g.,
Elavil) can be effective in relieving depression but may cause postural hypotension, fainting or confusion, thus
increasing fall risk. The elderly are particularly susceptible to adverse drug effects due to multitude of factors.

37. weakness and palpitations. Digitalis (Digoxin) is frequently used to treat congestive heart failure (it slows heart
rate and increases force of myocardial contraction). Adverse side effects of digitalis can include muscle weakness
and supraventricular or ventricular arrhythmias including ventricular fibrillation without premonitory signs.

38. a treadmill purchase. Any purchases of $300 or more would be considered a capital expense and listed as an
expense in the capital budget, not an operating budget.

39. wear gloves if there is direct contact with blood or body fluids. Standard Precautions specify health care workers
wear gloves when they come into direct contact with blood or body fluids. Health care workers should wear
moisture-resistant gowns and masks for protection from the splashing of blood, other body fluids or respiratory
droplets.

05 Sullivan A
36 Practice Questions

40. continuous monophasic current with the medication under the anode. Continuous unidirectional current flow is
more effective in repelling ions into skin than pulsed or bidirectional current. Procaine is a positive medicinal ion
and will be repelled from the anode (positive pole).

41. long term corticosteroid therapy. Very often patients with chronic pulmonary disease have been managed using
corticosteroid therapy. Long term steroid use affects ligamentous integrity, which often produces joint
hypermobility.

42. Medicaid Long term care for institutionalized elderly is funded by Medicaid, a state-federal partnership of funding.
Patients must first spend down their assets to qualify for low income stains before they are eligible for Medicaid.
Individual slates determine eligibility requirements.

43. infected pleura. The case is supportive of a pulmonary process as evidenced by radiography and history. Since the
radiographic findings and the pain are in the same vicinity, the likelihood is this pain is pleuritic in origin. Angina is not the
most likely cause since the cardiac system is not involved. There is no history of trauma to the chest and no trauma was
found radiographically, making it unlikely as the source of pain. An inflamed tracheobronchial tree would not usually
reflect pain in the posterior base of the left thorax.

44. training all staff to do simple repairs on all electrical equipment if a breakdown should occur. Electrical
equipment is repaired by the manufacturer or local vendor, or in some cases, the maintenance department, not by
physical therapy staff.

45. cellulitis. Cellulitis is an inflammation of the cellular or connective tissue in or close to the skin. It is characterized
by skin that is hot red, and edematous. Fever is a common finding. Dermatitis produces red, weeping, crusted
skin lesions, but is not commonly accompanied by fever. Location on shins makes herpes an unlikely choice and
there are no skin eruptions, or vesicles. Scleroderma is a collagen disease producing tight, drawn skin.

46. ultrasound. Ultrasound energy may affect the epiphyseal areas causing bone growth disturbances. The epiphyseal
plates close at the end of puberty.

47. 6-8 weeks of training. Hypertrophy is the increase in muscle size as a result of resistance training and can be
observed following at least 6-8 weeks of training. Individual muscle fibers are enlarged, contain more actin and
myosin, and have more, larger myofibrils.

48. external/lateral rotation and extension. The long head of the biceps is best exposed in shoulder lateral rotation
and extension due to its attachment at the supraglenoid tubercle of the scapula which is at the medial aspect of the
shoulder joint. Medial rotation and abduction places the long head of the biceps deep to the anterior deltoid and
pectoralis major muscles. The anterior surface of the shoulder, including the long head of the biceps, loses
exposure with horizontal adduction.

49. step up onto a low step while in the parallel bars. Active practice of stepping using a low step represents the best
choice to ensure motor learning. Passively bringing the loot up does not promote active learning. Choices A and C
are appropriate lead-up skills to stair climbing.

50. tandem walking and single limb stance. A patient with balance instability would benefit from all of the
interventions except tandem walking and single limb stance. These activities are too difficult (high level) to be
included in an initial training program. A significant percentage of healthy elderly are unable to stand on one leg.

51. 252 inches (21 feet). The architectural standard for rise of a step is 7 inches (steps may vary from 7-9 inches). The
recommended ratio of slope to rise is 1:12 (an 8% grade). For every inch of vertical rise, 12 inches of ramp will be
required. A straight ramp will have to be 252 inches or 21 feet long.

52. increased tidal volume. Asymmetrical breathing, deviated trachea towards the side to the pneumonectomy, and
decreased breath sounds would be likely changes. An increased tidal volume due to incisional pain and sedation
would not be likely.

53. shortness of breath at rest and with limited activity, and sudden weight gain. Elderly patients with moderate to
severe congestive heart failure demonstrate diminished cardiac reserve and intolerance to strenuous physical

05 Sullivan A
Practice Questions 37

activity. Symptoms of exertional intolerance include shortness of breath and sudden weight gain, the result of
circulatory backflow and peripheral edema.

54. position in prone lying and sitting with full knee extension. The typical contractures with a transtibial amputation
are knee flexion and hip flexion (typically from too much sitting). When in bed, hip and knee extension should be
emphasized (e.g. prone-lying). When sitting in the wheelchair, knee extension should be emphasized (e.g. using a
posterior splint or knee board).

55. postpone therapy and coordinate with the nurse regarding insulin management and exercise. Normal lasting
plasma glucose is less than I 15 mg/dL while a fasting plasma glucose level greater than 126 mg/dL on more than
one occasion is indicative of diabetes. This patient is hyperglycemic with high glucose levels (equal to or
greater than 250 mg/dL). Clinical signs that may accompany this condition include ketoacidosis (acetone breath)
with dehydration, weak and rapid pulse, nausea/vomiting, deep and rapid respirations (Kussmauls respirations),
weakness, diminished reflexes, and paresthesias. The patient may lie lethargic and confused and may progress lo
diabolic coma and death if not treated promptly with insulin. Physical therapy intervention is contraindicated;
exercise can lead to further impaired glucose uptake.

56. splinting the shoulder in abduction and internal rotation. Splinting the shoulder in abduction leads to formation of
abduction contractures and later hypermobility of the shoulder.

57. extension, left side-bending, and right rotation. The right sternocleidomastoid produces left lateral rotation and
flexion of the cervical spine. The right sternocleidomastoid is in lengthened position with the head turned to the right
and the cervical spine extended.

58. he should be expected to value patient confidentiality. Valuing and upholding patient confidentiality is an expected
behavior for all physical therapists and physical therapy students specified by the American Physical Therapy
Association Code of Ethics. Since this a final affiliation, the student should adhere fully to the APTA's Code of
Ethics.

59. B is less than A. In an A-B-A-B single subject design, the As represent multiple baseline-measurements while the Bs
represent multiple post-treatment measurements. If the hypothesis is accepted, the pain rating scores will he lower
following the treatment when compared to the baseline measurements.

60. improve rate control at faster movement speeds. Patients during the later stages of recovery from stroke
frequently exhibit problems with rate control. They are able to move at slow speeds but as speed of movement
increases, control decreases. An isokinetic device can be an effective training modality to remediate this problem.

61. use a shrinker. A shrinker is a suitable alternative to elastic wraps. It is important to select the right size
shrinker to limit edema and accelerate healing. An Unna's paste dressing is applied at the time of initial surgery.
Use of a temporary prosthesis should be a prosthetic team decision and is based on additional factors such as age,
balance, strength, cognition and so forth.

62. 5 times/week, BID 10 minutes/session. Patients with vascular insufficiency and claudication pain should be
encouraged to walk daily, 2-3 x/day. Duration should be short. The patient should walk to the point of maximum
tolerable pain, and allowed to rest.

63. oblique. The spinal defect is spondylolisthesis and the radiographic view that demonstrates the scotty dog neck
fracture is the oblique view. A lateral radiographic view will show the degree of anterior or posterior slippage of one
vertebra on another allowing the radiologist to grade the spondylolisthesis.

64. allow the patient to express her anger while refocusing her on effective coping strategies. Anger is a recognized
stage in the psychological adaptation to death and dying (Kubler-Ross). Patients should be allowed to express anger,
frustration, and resentment. A helpful strategy is to redirect the patient to achieve effective coping strategies (anger
management techniques). While honest, accurate information is important it is not the most useful strategy for this
patient at this time.

65. a facet joint. Facet joint dysfunction is exacerbated with sustained positions and eases with movement.

05 Sullivan A
38 Practice Questions

66. document the findings and immediately inform the patients physician about the situation. It is most appropriate to
discuss a patient problem with the person who is ultimately responsible for the patient. The physician as team
leader can affect a change in behavior or seek additional help for the patient.

67. forward step-ups in standing using graduated height steps. Decreased foot clearance during swing may result
from weak hip and knee flexors or may be the result of a drop foot (weak dorsiflexors or spastic plantarflexors).
Step-ups represents the best choice to functionally strengthen the hip and knee flexors. Bridging promotes knee
flexion with hip extension. The sitting activities promote hip abduction (A) and knee extension (B).

68. a reciprocating gait orthosis and walker. A patient with a SCI at the level of L1 or above has lost all
functioning lower extremity muscles needed for gait. A reciprocating gait orthosis is the best choice for this
patient. Because of the high energy expenditure, patients with injuries at this level typically depend upon the
wheelchair as their primary means of mobility. However, the physiological and psychological benefits make
it is appropriate to train this patient in ambulation.

69. appropriate if intensities are kept below 40% maximal voluntary contraction. Resistance training is typically
initiated after patients have completed 4 to 6 weeks of supervised cardiorespiratory endurance exercise. Lower
intensities are prescribed. Careful monitoring of BP is necessary as BP will be higher and HR lower than for
aerobic exercise.

70. change the angle of pull. Changing the angle of pull will reduce the traction pressure on the mandible and at the
TMJ. The greatest traction force should be felt on the occiput.

71. assisted breathing and coughing and pelvic floor exercises. It is important to teach the patient to provide incision
support for breathing and coughing. Pelvic floor exercises are also important since hours of labor and pushing arc
typically present before surgery. Assisted ambulation and ankle exercises would routinely be carried by nursing
personnel and would not necessarily require a P.T. referral.

72. spondylolisthesis with possible anterior slippage of the vertebral body. Spondylolisthesis is a bilateral fracture of
the pars interarticularis with anterior slippage. Spondylolysis is a unilateral or bilateral fracture without anterior
slippage.

73. adapt a desk and wheelchair to provide adequate sitting balance. The goal of school physical therapy is to directly
facilitate the educational process, for example, interacting in class viewing the blackboard, etc.

74. a rolling walker to compensate for impaired balance. The patient with Parkinson's disease typically presents with
postural deficits of forward head and trunk with hip and knee flexion contractures. Gait is narrow-based and
shuffling. A festinating gait may result from persistent forward posturing of the body near the forward limits of
stability. A rolling walker is contraindicated because it would increase forward postural deformities and festinating
gait.

75. ideomotor apraxia. With ideomotor apraxia, a patient cannot perform a task upon command but can do the task
when on their own. With ideational apraxia, a patient cannot perform the task at all.

76. should be paid for by a third party payer. Determination about whether care is paid for by a third party payer is
determined by the payer based on contract terms and condition. Ideally those terms include same or similar clinical
factors and rationale that are part of appropriate peer review but they need not be.

77. increased total lung capacity. An obstructive pattern on pulmonary function tests includes increased total lung
capacity caused by destruction of alveolar walls. This same destruction causes an increased residual volume with a
resulting increased functional residual capacity and decreased vital capacity.

78. ischial tuberosity, gluteals, and lateral sides of residual limb. A quadrilateral socket in a transfemoral amputation is
designed to selectively load tissues that are pressure tolerant: the ischial tuberosity, gluteals, and lateral sides of the
residual limb.

79. immediately contact her primary physician. This patient is presenting with signs of clinical depression. Her primary
physician should be contacted immediately (especially if suicide is mentioned).

05 Sullivan A
Practice Questions 39

80. mechanical stretching using traction and 5 lb. weights, 2 hours, twice daily. Prolonged mechanical stretching
involves a low-intensity force (generally 5 to 15 Ibs.) applied over a prolonged period (30 minutes to several hours). It is
generally the most effective way to manage long-standing flexion contractures. Manual passive stretching is a short duration
stretch that is not likely to be effective in this case. Lower extremity splints and hold-relax techniques can be used to improve
flexibility but are also not likely to be effective in this case because of the short duration.

81. stretching and limiting extended spinal positions as the result of spinal stenosis. Spinal stenosis presents with
bilateral dysesthesias and pain in extended positions and/or during walking for distances greater than 100 feet.
Physical therapy intervention should emphasize stretching of tight structures and dynamic control of the trunk to
limit long-term extended spinal positions.

82. have the teacher give a smile sticker when the child sits with head retracted for five minutes. Positive
reinforcement (use of the smile sticker) is an effective way to shape behavior by operant conditioning. Negative
behaviors are ignored (slumping in the chair) while positive behaviors are encouraged (hold head in a retracted
position for 5 minutes.

83. ask your colleagues about their current level of knowledge using a brief questionnaire. In order to better share the
information you have learned, you need to assess what information and skills your colleagues currently have. A
brief questionnaire can be an effective means to achieve this. Choices A, B, C, demonstrate planning of the
learning experience without benefit of a needs assessments.

84. passive manipulation to the shoulder. All of the treatments may lie used for RSD (Complex Regional Pain
Syndrome, CRPS) except passive manipulation which may aggravate sympathetically maintained pain.

85. lower trapezius, latissimus dorsi, and triceps. The upper quadrant muscles that arc most important to
strengthen for crutch gaits include the lower trapezius, latissimus dorsi, and triceps. Shoulder depression and
elbow extension strength is crucial.

86. 92%. A 92% SaO2 is low but acceptable. Unacceptable oxygen saturation rates during exercise are <
88%. Normal SaO2 is >95% - 98%.

87. ice massage B.I.D. until the pain subsides. Pain and inflammation can be effectively treated with a cold modality.
Ice massage can rapidly reduce pain and tissue temperature in a small area without affecting the bony structure.
Treatment frequency should increase if pain is severe. Fluidotherapy three times a week is not appropriate.

88. side-bending. The uncinate processes (joints of Luschka) are located-at the inferior lateral aspect of the lower
cervical vertebrae. Side-bending is lost with degenerative changes at the joint the uncinate process makes
with the vertebra below. Other motion is restricted but to a lesser degree.

89. active assistive pulley exercises. The patient is most likely suffering from a rotator cuff tear. Acute physical
therapy intervention should focus on reduction of pain and inflammation. During the early subacute phase,
active assistive pulley exercises would be indicated lo promote healing of the supraspinatus muscle and
maintain active range of motion of the glenohumeral joint.

90. interrater reliability. Interrater reliability is the degree to which two or more independent raters can obtain the
same rating for a given variable. In this case, two therapists obtained different HIM scores for the same group of
patients, indicating a problem in interrater reliability.

91. sit the patient up, check/empty catheter, and then call for emergency medical assistance. The patient is exhibiting
autonomic dysreflexia (an emergency situation). You should first sit the patient up and check for irritating or
precipitating stimuli (e.g., a blocked catheter). Then call for emergency medical assistance.

92. metatarsal bar. Correction for a pes planus deformity (support of the longitudinal arch) can be provided by a
UCBL insert, scaphoid pad or Thomas heel. A metatarsal bar is indicated to take pressure off the metatarsal heads
and improve push off.

93. custom made pressure garments. Following burns, edema and hypertrophic scarring can be effectively controlled
with elastic or pressure garments. Surgery is an option of last resort.

05 Sullivan A
40 Practice Questions

94. dyspnea, anxiety, or disorientation. A patient with respiratory acidosis may present with symptoms of dyspnea,
anxiety, restlessness, or headache. Significant acidosis may lead to disorientation, stupor, or coma. Answers A, B,
C are signs and symptoms of respiratory alkalosis.

95. placing the wrists in resting splints. Modalities which use steroids are contraindicated for pregnant women. The most
effective intervention would be to place the wrists in a neutral position in splints. The carpal tunnel is, therefore, not
compromised by poor hand positioning while at work.

96. wheelchair with a back wedge and head supports. A wheelchair wedge and head supports holding the trunk and
head in slight flexion will help decrease the extensor tone and is the most appropriate positioning in the
educational setting. An abduction pommel controls scissoring of the legs but does not control head and upper trunk
necessary for functioning in the school environment.

97. shoulder extensors, external rotators, and anterior deltoid to position and lock the elbow. The patient with
complete C^, quadriplegia will lack triceps. He can be taught to lock the elbow for push-up transfers by using
shoulder external rotators, extensors to position the arm; the anterior deltoid locks the elbow by reverse actions (all
of these muscles are functional).

98. demonstration, practice, and follow-up discussion. A variety of teaching methods including demonstration,
practice, and discussion has the best chance of reinforcing the learning in a diverse group. One teaching method is
not likely to be as successful in meeting the needs of all group members.

99. have the patient wear a tight fitting mask while being treated in his room. The therapist should wash his/her hands
upon entering and leaving every patients room. When the patient is suspected of having tuberculosis, the patient
should be in a private, negative pressurized room. The room is considered a potentially infective environment and
the therapist should don a tight fitting mask prior to entering the room. The patient only needs to wear a mask if he
needs to leave his room (for a medical test, etc.). See Transmission-Based Precautions, Table 5-3.

100. patient education regarding avoidance of squatting and jumping activities as well as initiation of iontophoresis
using dexamethasone. The dysfunction observed on the x-ray is Osgood-Schlatter's Disease. The radiograph
depicts epiphysitis of the tibia at the attachment of the patellar tendon seen in adolescents. It occurs as the result
of activities that require continued explosive contractions of the quadriceps muscle complex during pubescent
growth spurts. Patient education should locus on controlling knee loading activities such as squatting and
jumping. Explosive contractions of the quadriceps complex should be avoided. Ambulation and AROM
activities maintain mobility while the structure heals, Phonophoresis would be con traindicated because it may
be painful to move the sound head over the affected area. Additionally, ultrasound should not be used over
open epiphyses. Open chain knee extension exercise may aggravate symptoms due to the increased load at
the attachment of the patellar tendon to the tibial tuberosity.

101. provide a two-wheel handcart for use in moving the boxes. Implementation of an engineering control
technique can be accomplished by designing or modifying the workstation, work methods, and tools to
eliminate/reduce exposure to excessive exertion, awkward postures and repetitive motions.

102. subtalar pronation and medial rotation of the tibia. In order to maintain the center of gravity over the base of
support, the subtalar joint must pronate and the entire lower quarter must medially rotate.

103. empty end-feel. An empty end-feel (no real end-feel) may be indicative of severe pain and muscle guarding
associated with pathological conditions. Springy and firm end-feels may be expected after elbow surgery. Soft end-feel
is an indication of range limited because of tissue compression (e.g. in knee flexion there is contact between the
posterior leg and the posterior thigh).

104. behind and to the left side, one hand on the gait belt. The correct guarding technique to protect a patient from
failing is to stand slightly behind and to one side (the involved or left side).

105. unaffected. The chronaxie is the duration of the current at twice the rheobase (minimal intensity at a given long
duration). The chronaxie of an innervated nerve is less than then 1 ms. When a motor nerve is severed it undergoes a
process called Wallerian degeneration in which the axons distal to the lesion degenerate. The chronaxie of the
denervated nerve is greater than I ms. During the first 7 to 10 days much of the nerve constituents are st il l present,
thus no change in the chronaxie is observed.

05 Sullivan A
Practice Questions 41

106. swimming using a crawl stroke. Positioning in spinal extension increases symptoms in patients with spinal stenosis.
Activities such as swimming using a crawl stroke place the spine in a constant extended position. All other
activities described do not require the patient to maintain an extended spinal position.

107. a shoe lift on the sound side. This patient is vaulting (rising up on the sound limb to advance the orthotic limb
forward). This gait deficit increases the energy demands of gait and is most likely contributing to his fatigue. The
myalgia and fatigue arc also direct impairments of post-polio syndrome. An appropriate intervention plan would be
to reduce the energy demands of gait. The use of a shoe lift on the sound side would help accomplish this. A
wheeled mobility device (motorized cart) would also assist in reducing energy demands of functional mobility. The
wheelchair choices presented are poor ones.

108. stimulate the ipsilateral triceps. Stimulation of the ipsilateral triceps will reduce the muscle tone of the biceps
through reciprocal inhibition.

109. ankle-foot orthosis. Individuals with an L5 lesion need an orthosis to correct foot alignment and for push-off.
There is partial innervation of the hamstrings and gluteus minimus and medius, posterior tibialis and peroneus
tertius muscles. The other orthoses would he indicated for higher level lesions.

110. closed-chain partial weight-bearing lower extremity exercises for slipped capital femoral epiphysis. Slipped
capital femoral epiphysis is characterized by a glutens medius gait. Closed-chain exercises with weight-bearing to
tolerance will help regain or maintain functional muscular strength and normal motion.

111. null hypothesis. The null hypothesis is a statistical hypothesis that states that there is no relationship (or
difference) between variables. Any relationship found will be a chance relationship, not a true one.

112. that the therapist was functioning outside the common protocols of the hospital, and therefore did not support the
actions of the physical therapist. The physical therapist was acting outside of her area of responsibility and did
something that caused the patient harm.

113. patch one eye. Double vision (diplopia) can be managed by patching of one eye. Patients are typically on an eye
patching schedule which alternates the eye that is patched. Loss of depth perception can be expected with eye
patching but is not as disabling as diplopia.

114. sidelying on the sound side with the affected upper extremity supported on a pillow with the shoulder protracted
and elbow extended. Most patients with stroke recover from the flaccid stage and develop spasticity. Positioning
for the patient with early stroke stresses (I) protection against ligamentous strain and the development of a painful
subluxed shoulder and (2) positions counter to the typical spastic posture of flexion and adduction with pronation.
Sidelying with the affected upper extremity supported on a pillow with the shoulder protracted and elbow extended
accomplishes both of these goals. The other positions do not.

115. chronic venous insufficiency. Venous ulcers are often painless, or present with minimal pain when compared to arterial
ulcers which are painful (claudication and rest pain). Chronic venous insufficiency is also characterized by
thickening, coarsening, and brownish pigmentation of the skin around the ankles. The skin is usually thin, shiny,
and cyanotic. DVT may be asymptomatic initially. When symptoms occur, patients typically report a dull ache,
tightness, or pain in the calf.

116. large electrodes, widely spaced; 10:30. Large electrodes are used on large muscles in order to disperse the current
(minimize current density under the electrode) enabling a more comfortable delivery of current. Widely spaced
electrodes permit the current to travel deeper into the muscle to stimulate a greater number of deeper muscle
fibers. A duty cycle of 1:3 will allow a sufficient rest period between contractions to minimize the fatiguing effect
of repetitive muscle contractions.

117. call the physician immediately and report your findings. A mole with irregular edges and multiple colors (black or
blue) is characteristic of a melanoma. It is associated with sun-damaged skin but can appear anywhere on the
body. Early recognition before tumor invasion and metastatic spread is critical in improving survival rates. The
physician should be informed immediately. Medical management consists of biopsy and surgical removal.

05 Sullivan A
42 Practice Questions

118. decreasing consciousness with slowing of pulse and Cheyne-Stokes respirations. Signs of increased intracranial
pressure secondary to cerebral edema and brain herniation include decreasing consciousness with slowing of pulse and
Cheyne-Stokes respirations. Cranial nerve dysfunction is typically noted in C.N. II (papilledema) and C.N.III
(dilation of pupils). Choices A and C are signs of meningeal irritation and CNS infection.

119. nerve dysfunction will be rapidly reversed, generally in 2-3 weeks. Neurapraxia is a mild peripheral nerve injury
(conduction block ischemia) that causes transient loss of function. Nerve dysfunction is rapidly reversed,
generally within 2 - 3 weeks. An example is a compression injury to the radial nerve from falling asleep with
the arm over the back of a chair (Saturday night palsy).

120. occasional feedback given when consistent errors appear. In learning a psychomotor skill, the patient needs to be
able to actively process information and self-correct responses. Occasional feedback provides the best means of
allowing for introspection and is appropriate for later in practice (associated and autonomous phases of motor
learning).

121. he may be presenting with early signs of myocardial infarction. An elderly patient with a cardiac history may
present with initial symptoms of mental confusion, the result of oxygen deprivation to the brain. The SOB and
generalized weakness may also be due to generalized circulatory insufficiencies coexisting with the developing
myocardial infarction.

122. modalities to reduce inflammation, active assistive range of motion exercises using pulleys, and postural
realignment. Initial physical therapy intervention of tendinitis with possible impingement should emphasize active
assistive exercises, modalities to reduce inflammation, and postural realignment to reduce impingement of the
rotator cuff at the acromion.

123. 10 to 15 seconds. The recommended time duration for endotracheal suctioning is 10 to 15 seconds. Any longer
risks serious any shorter and the risk is ineffective secretion removal.

124. occlusive dressings. Autolytic wound debridement allows the body's natural enzymes to promote healing by
trapping them under a synthetic, occlusive dressing. The dressings arc applied for short durations (less than 2
weeks ) and arc contraindicated in infection. The other answer are appropriate wound management techniques;
however, they are not autolytic.

125. switch to modulation mode TENS. Because of the long-term continuous use of TENS, the sensory receptors
accommodated to the continuous current and no longer responded to the stimuli. Changing to modulation mode (i.e.
burst modulation), which periodically interrupts the current flow docs riot allow accommodation to occur.

126. gastrocnemius-soleus. The muscles of the foot and ankle move the long lever of the body forward and backward
using ankle synergies. The gastrocnemius-soleus moves the body backward while the anterior tibialis moves the
body forward. Action of the hip extensors would result in a backward lean with the center of motion occurring at
the hip.

127. false negative. Sensitivity refers to the ability of a test to correctly identify individuals who truly have a disease or
condition (a true positive). In this example, the individual was found not to have the Alzheimer's gene, when years
later he tested positive for the gene and the disease. The original test produced a false negative result.

128. history of sudden onset of new cognitive problems and patchy distribution of deficits. Multi-infarct dementia
differs from primary degenerative dementia, Alzheimer's type, in (1) onset: sudden rather than slowly progressive
(characteristic of small focal infarcts) and (2) the nature of symptoms: areas of deficits coexist with areas of intact
cerebral function. Agitation and sundowning (late afternoon wandering) are characteristics of Alzheimer's disease.

129. vastus medialis muscle strengthening. Q angles greater than 15 degrees could be indicative of abnormal lateral
patellar tracking. Vastus medialis muscle strengthening can reduce the tendency for the patella to track laterally.
Vastus lateralis (VL) strengthening can promote greater lateral patellar tracking and further irritation of the
patellofemoral joint. VL strengthening may promote an outward pull or dislocation of the patella. Hamstring
strengthening does not directly affect tracking of the patella.

130. standing, picking the foot up behind and slowly lowering it. Stage 4 recovery is characterized by some movement
combinations that do not follow that paths of either flexion or extension synergies. Knee flexion in standing is an out-

05 Sullivan A
Practice Questions 43

of-synergy movement. All other choices represent synergistic movements: choices A and B are flexion synergy
movements while choice C focuses on knee extensor movement within an extended position.

131. examine the patient, document and discuss your findings with the doctor. The physical therapist should complete
the examination of the patient, adequately document the findings, and determine the physical therapy diagnosis.
While many states have direct access laws which permit physical therapy intervention without referral, most
insurance companies including Medicare (affecting the patient in this example) require a physician referral in
order for services to be reimbursed. Thus the therapist needs to consult with the physician to get a referral before
initiating any intervention for this problem. This patient demonstrates stress incontinence, a problem that could be
successfully treated with physical therapy (e.g., Kegel exercises and other interventions).

132. anterior spinothalamic tract. Sensations interpreted as dull, aching pain travel in the anterior (paleo)
spinothalamic tract. Discriminative fast pain is carried in I he lateral (neo) spinothalamic tract. Discriminative touch
is carried in the proprioceptive pathways (fasciculus gracilis/cuneatus, medial lemniscus).

133. increase the patients maximal oxygen consumption by discharge. All of the choices are important goals for Phase
I, acute or inpatient cardiac rehabilitation except for altering maximal oxygen consumption. Patients generally are
expected to reach a functional capacity of 3-5 METs by discharge from a Phase I program.

134. placement of the drive wheels 2 inches posterior to the vertical back supports. Placement of the drive wheels 2
inches posterior to the vertical back supports is an appropriate modification for a patient, with bilateral
transfemoral amputations. This increases the length of the base of support and provides increased posterior stability.
Lowering the seat height by 3 inches is an appropriate modification for a patient with stroke who will use his
sound limbs for wheelchair propulsion. Increasing the seat depth is not an appropriate modification.

135. P=0.05. A preselected probability level of 0.05 indicates that the results (in this example differences in
measurements) would be the result of chance only 5 times out of every 100 studies. This level of confidence
helps us reject the null hypothesis (there is no difference in goniometric measurement techniques) and is common
in most experimental studies (0.01 is the other, more stringent level of significance commonly applied).

136. instruction in proper postural alignment. Without regaining normal postural alignment and scapular-humeral
rhythm, the patient will continue to impinge the supraspinatus and/or biceps tendon at the acromion and never
regain normal function of the shoulder. It is unlikely that all pain would be controlled.

137. pain and stiffness are worse in the early morning and should decrease with moderate activity. Degenerative joint
disease (osteoarthritis) is a noninflammatory progressive disorder affecting primarily the carpometacarpal, knee, and hip
joints. Stiffness is common following inactivity and is relieved with movement. Exercise can be helpful in reducing
pain and immobility and is not contraindicated. Joint projection strategies are effective intervention for maintaining
function.

138. symmetrical distribution of weakness, ascending with possible involvement of lower cranial nerves. Guillain-
Barre syndrome is an acute polyneuritis characterized by rapid development of progressive muscle weakness.
The weakness is typically symmetrical and ascends the body (starting first in the lower extremities, progressing to
trunk, upper extremities, and finally cranial nerves). Stocking and glove sensory loss could also be found but not
with minor loss of motor function.

139. monophasic current with the anode placed on the tendon. The anode (positive) electrode would effectively repel
the positively charged ion into the tendon. Because of its unidirectional flow, monophasic currents are best suited
for iontophoresis.

140. have the patient do sitting push-ups at least every 10 minutes. Excessive ischial pressure and redness from
prolonged sitting requires an aggressive approach. Arm push-ups, at least every 10 minutes, is indicated if redness
is present. A high-density gel cushion might help as would a tilt-in-space chair but neither of these would address
the immediate problems.

141. medial rotation. Common abnormal postural findings consistent with anterior knee pain in an adolescent female
include pes planus, lateral tibial torsion and genu valgum with compensation at the femur of excessive medial
rotation. Lateral femoral rotation is commonly observed with genu varum. Retroversion of the hip is an abnormally
small angle between the femoral neck and condyles and is not affected by posture.

05 Sullivan A
44 Practice Questions

142. you will meet with the occupational therapist to discuss the exercise approaches and you will let them know the
outcome of the meeting as soon as possible. Any discrepancies in educational approaches used with the patient's
family should be handled by the learn members, not the family or rehabilitation supervisors. A team meeting is a good
place to iron out approaches to patient care and ensure consistency.

143. a videotape of another child with cerebral palsy on a Swiss ball. A videotape of another child with CP on the ball
represents the best choice to engage this child: The other choices, while important educational media, are not likely
to adequately present the three-dimensional qualities of performance needed for exercising on the Swiss ball.

144. summon emergency medical services. If the victim is unresponsive, Emergency Medical Services must be
activated immediately by calling 911 or calling a "code" if in a health care institution. If the victim is a young child
or infant, try one minute of rescue breathing before summoning help.

145. call the girls pediatrician immediately. These signs could be the result of increased cerebral edema due to a
clogged or infected shunt. Medical attention should he obtained immediately to avoid damage to the brain.

146. protection and splinting of the abdominal musculature. Diastasis recti abdominis is a condition in which there is a
lateral separation or split of the rectus abdominis. It is important to teach protection (splinting) of the abdominal
musculature. Patients should be instructed to avoid full sit-ups or bilateral straight leg raising. Pelvic floor
exercises should be done but are not remediation for diastasis recti.

147. supine, head of bed flat. A patient with a C5 spinal cord injury will not have the abdominal musculature
necessary to return the diaphragm to a high domed position during exhalation. Inspiration will be affected by the
change in the diaphragm's resting position. In supine, gravity will take the place of abdominals, holding the
abdominal contents under the diaphragm, improving the zone of apposition, the height of the diaphragm dome
and therefore, the ability to ventilate. The other positions listed negate the positive effects of gravity on the
abdomen.

148. schedule a conference with the doctor and family about her condition and your discussions with the patient. The
most appropriate strategy is to hold a conference with the doctor and family about her condition and your discussions
with the patient. Everyone interacting with this patient should be answering her questions in the same way. A direct
and honest approach is best but must be consistent with the parents wishes since she is a minor child.

149. overwork damage in weakened, denervated muscle. Amyotrophic lateral sclerosis is a progressive degenerative
disease that affects both upper and lower motor neurons. An important early goal of physical therapy is to
maintain the patient's level of conditioning while preventing overwork damage in denervated muscle (lower
motor neuron injury). Myalgia is common in I,MN lesions. It can be ameliorated " but not prevented. Ataxia and
radicular pain are not associated with ALS.

150. slight wrist extension with fingers supported and thumb in partial opposition and abduction. A resting splint that
positions the wrist and hand in a functional position includes 10-20 degrees of wrist extension, fingers supported, and
thumb in partial opposition and abduction.

151. complaints of fatigue with increasing dyspnea and cough. Left heart failure is a condition in which blood is not
adequately pumped out of the heart by the left ventricle. The blood; therefore, backs up creating pulmonary signs
and symptoms (increased pulmonary artery pressures, cough, dyspnea, orthopnea), weakness, and fatigue. All
other choices are signs associated with right-sided heart failure.

152. take a rest period and only use the device 10 times per hour. If a patient feels lightheaded with an incentive spirometer,
it may be that they are blowing off too much CO2 by hyperventilating. An incentive spirometer should always be used in
the most upright position possible to attain the highest values possible.

153. 30 minutes each day. Motor learning is enhanced by daily practice of moderate duration.

154. compression of the long thoracic nerve. Vigorous upper limb activities can cause inflammation of soft tissues
surrounding the shoulder resulting in compression of the long thoracic nerve and weakness of the serratus anterior. The
serratus anterior stabilizes the scapula with greater than 90 degrees of abduction. Supraspinatus tendinitis or weakness

05 Sullivan A
Practice Questions 45

does not result in scapular winging. The supraspinatus muscle in concert with the deltoids initiates abduction in the upper
extremity. Subdeltoid bursitis causes pain with all active range of motion arid does not result in scapular winging.

155. sacral sitting. Spasticity is typically strong in antigravity muscles. In the lower extremities this is usually the hip and
knee extensors, adductors, and plantar flexors. Strong extensor tone results in sacral sitting with the pelvis tilted
posteriorly.

156. peripheral edema. Peripheral edema is a symptom of overdosage/toxicity for patients on Premarin. The risk of
osteoporosis and fracture is reduced for individuals on estrogen replacement therapy. Control of hot flashes is a major
indication for its use.

157. jogging is safe but you might want to switch to swimming during later months. According to the American
College of Sports Medicine, women can continue to exercise regularly (3 times a week) throughout pregnancy
if no additional risk factors are present. After the first trimester women should avoid exercise in the supine position
since this position is associated with decreased cardiac output. Non-weightbearing exercise (swimming) is an
appropriate alternative to decrease the risk of injury and facilitate continuation of exercise throughout pregnancy.

158. concentric exercises, 3 sets of 10, with gradually increasing intensity. This patient is experiencing delayed-
onset muscle soreness (DOMS) as a result of vigorous exercise or muscular overexertion. It typically
begins 12 to 24 hours after exercise, peaks in 24 to 48 hours, and can last up to 5 to 7 days. DOMS is
usually greater after muscle lengthening or eccentric exercise and can be lessened by gradually increasing
intensity and duration of exercise.

159. an iliotibial band friction disorder. A positive Noble compression test is an indication of an i l i o t i b i a l band friction
disorder. A FABERE test can distinguish between h i p joint pathology and sacroiliac joint dysfunction.

160. problems with motor planning and scaling of movements. The basal ganglia functions to convert general motor
activity into specific, goal-directed action plans. Dysfunction results in problems with motor planning and scaling
of movements and postures (e.g., bradykinesia).

161. vagus nerve. These are the tests for the vagus nerve C.N. X. See chapter 2 for cranial nerve tests.

162. increased volume of air at resting end expiratory pressure (REEP). The gravity independent area of the lung in the
upright sitting position refers to the apices of I he lungs which house the most air at REEP. That area has the least
perfusion because of the effects of gravity on blood flow. The apices also have the smallest change in ventilation
during the respiratory cycle because they are the most full at rest. Finally, this area of the lung has the highest
oxygenation and lowest carbon dioxide content due to the small volume of blood that needs to be diffused.

163. just anterior and superior to the greater trochanter. The orthotic hip joint should coincide with a point just anterior
and superior to the greater trochanter (the anatomic center of the hip joint).

164. the PTA who is completely liable because the plan of care was altered without communicating with the supervising
PT. Only the supervising PT may alter the established plan of care. In this case the treatment plan stipulated progressive
ambulation on level surfaces only. Therefore, the PT is not responsible, the PTA is responsible and negligent.

165. gentle AROM exercises in weight bearing. Aggressive soft tissue stretching and strengthening can promote myositis
ossificans. Gentle weight bearing AROM exercises to patient's tolerance will minimize the chance of myositis
ossificans and promote improved function.

166. a tone inhibiting ankle-foot orthosis (AFO). A tone inhibiting AFO will make ambulation easier and will help
maintain ROM of the affected ankle. The most appropriate assistive device would be a cane (i.e. straight or SBQC),
not a walker.

167. inadequate contraction of the ankle dorsiflexors. Weakness or delayed contraction of the ankle dorsiflexors or
spasticity in the ankle plantarflexors may cause foot drop during midswing. Excessive extensor synergy would
cause plantarflexion during stance.

168. tell her that she must have the permission of her father before she can look at the chart. The only time
confidentiality can be breached is if the patient agrees or he is in danger. Even the family doesn't have the right to the

05 Sullivan A
46 Practice Questions

medical information without the patient's permission unless the patient is a minor child or is incompetent to make
decisions.

169. decrease edema. The reduction of edema is through the diffusion of synovial fluid. In addition, increased venous flow and
increased nutrition is occurring in and around the joint enhancing the healing process. The reduction of pain is a
secondary consequence of decreased inflammation. Range of motion is usually limited during the first few hours of
treatment and no tension is to be placed on the surrounding tissues.

170. five times per week. The patient with acute tendinitis receiving iontophoresis should come daily the first two weeks to
enable optimal amounts of medication to affect the inflamed tissue. The patient needs to return to work as soon as
possible.

171. hip flexion contracture. A hyperextended knee can be caused by extensor spasticity, quadriceps weakness (a
compensatory locking of the knee), or by plantarflexion contractures or deformity.

172. call emergency medical technicians as it may be a pneumothorax. The combined signs and symptoms of absent
breath sounds, sudden onset of chest pain and shortness of breath indicate a pneumothorax, especially in a 15
year-old male (growth spurt) with pathological changes of king tissue. This should be considered an emergency
situation.

173. supervision of home health aides for completion of a daily home exercise program. Medicare funds home health
care under Part A. Teaching of home care aides to ensure completion of a daily home exercise program is a
reimbursable service while routine provision of an established maintenance program in which the therapist performs
the exercises would not be covered.

174. PNF bilateral symmetrical upper extremity D2 flexion patterns, rhythmic initiation. The patient with Parkinson's
disease typically develops elbow flexion, shoulder adduction contractures of the upper extremities along with a
flexed, stooped posture. Bilateral symmetrical upper extremity PNF D2F patterns encourage shoulder flexion and
abduction with elbow extension, and upper trunk extension (all needed motions). Both quadruped and modified
plantigrade positions encourage postural flexion.

175. perform the ROM exercises yourself. The practice of using supportive personnel falls under the Guide for
Professional Conduct (Appendix B0 and under the individual practice acts of the states. Delegated responsibilities
should be commensurate with the qualifications (experience, education, training) of the individual to whom
responsibilities are being assigned. In this case, it is not reasonable to assume an aide newly arrived to the P.T.
department has the knowledge or skills to do this treatment. A brief orientation to ROM exercises is not adequate
to ensure proper treatment. The therapist should do the ROM exercises herself. In some states, any treatment by an
aide would be inappropriate.

176. wheelchair with head and trunk support with abductor wedge. The wheelchair would be the most functional
adaptive equipment with postural support and abductor wedge to help decrease extensor spasms of lower
extremities. This individual is not stable enough to use a rollator walker or scooter board.

177. back extension, costal expansion exercises, and maintenance of proper posture to prevent deformity. Postural
reeducation will help to prevent further increase in thoracic kyphosis and costal expansion exercise will improve
breathing efficiency. Abdominal strengthening and stretching of scapular stabilizers are not indicated for
ankylosing spondylitis.

178. contralateral hemiparesis and sensory deficits with greater involvement in the arm than the leg. A CVA affecting
the middle cerebral artery will result in symptoms of contralateral hemiparesis and hemi sensory deficits with
greater involvement of the arm than the leg. Choice B is incorrect since involvement would be the left hemisphere
with right hand dominance. Choice C (greater leg involvement than arm) is characteristic of a CVA affecting the
anterior cerebral artery.

179. elevated levels of low-density lipoprotein or LDL. Increased blood cholesterol levels with high levels of low-
density lipoproteins (LDLs) increases the risk of coronary artery disease (CAD); conversely low
concentrations of high-density lipoproteins (HDLs) is also harmful. The link between CAD and
triglycerides is not as clear.

05 Sullivan A
Practice Questions 47

180. thank him very much, but refuse his invitation while he is receiving treatment. Physical therapy departments should
have a policy stating it would be unethical to have sexual contact or dating between staff and patients. The Guide
for Professional Conduct of the American Physical Therapy Association (Appendix B, Administration Chapter)
states in Principal 2 that physical therapists shall not engage in any sexual relationship or activity, whether
consensual or non-consensual with any patient while a physical therapist-patient relationship exists. Some authorities
suggest a waiting period of 6 months after discharge from treatment before commencing an intimate relationship.

181. referral to a physician to evaluate anterior compartment pressures during activity. Deep anterior leg cramping
after running distances- one mile or greater would most likely be due to a local ischemic condition of the anterior
compartment as a result of excessive pressure. Shin splints and tendinitis symptoms would occur earlier in running
and be felt as pain. Referral to a physician is indicated if nontraumatic compartment syndrome is suspected.
Traumatic anterior compartment syndrome requires immediate attention by a physician. Spinal stenosis usually
manifests itself as posterior leg cramping.

182. teach her activity pacing and energy conservation techniques. You should teach this patient activity pacing and
energy conservation techniques. It is important she is able to balance rest with activity in order to not further
weaken muscles affected by progressive post polio muscular atrophy.

183. ligamentum flavum. The ligamentum flavum becomes hypertrophied with lumbar spondylosis and may invade the
intervertebral foramen compressing the left L5 spinal nerve root.

184. onset of moderate to severe angina. According to the American College of Sports Medicine, an absolute
indication for terminating an exercise bout is onset of moderate to severe angina. Other absolute indications
include: acute MI, a drop in SBP with increasing workload, serious arrhythmias (2nd or 3rd degree heart blocks,
sustained ventricular tachycardia or PVCs, atrial fibrillation with fast ventricular response), unusual or severe
shortness of breath, CNS symptoms (ataxia, vertigo, confusion), or patient's request. The other choices are
considered relative indications and require close monitoring.

185. notify the physician immediately, you suspect heterotopic ossifications may be developing. Early signs of
heterotopic bone formation include swelling, pain, erythema occurring near a large joint. The physician should be
notified immediately. Vigorous stretching is contraindicated.

186. clearance between the floor and the foot plate of at least 2 inches. The correct measure for seat height in a
wheelchair is 2 inches clearance between the floor and the foot plate, measured from the lowest point on the
bottom of the footplate.

187. recreational activities such as swimming or biking. Recreational exercise will be both fun and helpful in
maintaining functional level as long as possible. At 6 years of age, wheelchair confinement is not usual.

188. currently there is no limit to her Medicare coverage for outpatient PT services. From September J, 2003 through
December 7, 2003 there was a limit on the amount Medicare would pay for outpatient physical therapy services.
The limit of $1590.00 was in effect for outpatient PT services given at a private clinic but not for hospital
outpatient department services. Effective December 8, 2003 the limit was removed by the recent Medicare Prescript
ion Drug and Modernization Act of 2003. Currently there is no limit to the amount of medically necessary outpatient
PT, OT, or SLP services.

189. use an alternate eyelid taping schedule which will allow her to use eye movements to communicate. Pontine
lesions that result in locked-in syndrome leave the patient with an inability to move or speak but with full cognitive
function. By taping one eye open, the patient's ability to receive sensory inputs is increased (sensory deprivation is
lessened) and eye movements can be used for communication. They should not expect any active responses from the
patient.

190. maintain the keyboard in a position allowing a neutral wrist position. Work involving increased wrist deviation
from a neutral posture in either flexion/extension or radial/ulnar deviation have been associated with increased
reports of carpal tunnel syndrome and other wrist and hand problems.

191. depressed ST segment with a flat T wave and shortened QT interval. Digitalis produces characteristic changes on
the ECG: gradual downward sloping of ST segment with a flat T wave and shortened QT interval.

05 Sullivan A
48 Practice Questions

192. intermittent pneumatic compression, extremity elevation, and massage. Lymphedema following surgery and
radiation can be effectively managed by external compression and extremity elevation. Manual lymph drainage
(massage and PROM) are also appropriate interventions. Exercise and positioning alone would not provide the
needed lymph drainage; isometric exercise is contraindicated.

193. suggest that the patient speak with his surgeon. It is within the physician's scope of practice to discuss the
indications and problems that could arise from this surgical procedure. It is not within the physical therapist's scope of
practice to be the expert who discusses problems associated with surgery.

194. gravity assisted right upper extremity exercises to promote scapular control following damage to the long thoracic
nerve. With axillary dissection the long thoracic nerve may be damaged. This leads to serratus anterior
weakness and loss of scapular control. Gravity assisted exercises to promote scapular control should be
emphasized early in rehabilitation to help restore proper scapular humeral rhythm.

195. amount of productive secretions decreases. The purpose of postural drainage is to help remove secretions. If
the amount diminishes this might be an indicator that the treatment has been successful and that the frequency
of treatment can be reduced. The other choices of lever, sputum consistency and pain do not provide a rationale
to decrease treatment frequency.

196. chronic, episodic bouts of dysfunction. Meniere's disease is a recurrent and usually progressive vestibular
disease characterized by episodic bouts of dysfunction. As w i t h most vestibular disorders, varying degrees of
vertigo, nausea, and postural i n s t a b i l i t y will occur.

197. Workers Compensation. T h e employee was injured on the job even though she arrived early. She is therefore
covered by Worker's Compensation for her injury.

198. postural reeducation to prevent positions that increase symptoms. Patients should be educated to prevent extended
positions for long periods. Symptoms resulting from Paget's disease are aggravated by positions where the lumbar
spine is in extension. Because this is a chronic condition, modalities are not the most effective management
strategy. Lumbar extension exercises decrease the space within the vertebral foramen thereby increasing symptoms
associated with stenosis and Paget's disease.

199. the posterior stop setting the foot in too much plantarflexion. An ankle-foot orthosis can be used to assist in knee
control for the patient with hemiplegia. A foot set in slight plantarflexion (generally about 5 degrees) helps to
create an extension moment at the knee and can thus assist weak knee extensors in extending the knee. However if
the ankle is set in too much plantarflexion, it can cause genu recurvatum.

200. daily walking through pain in order to attain predetermined distances. Patients with peripheral vascular disease
with intermittent claudication can benefit from a walking program to improve peripheral blood flow and functional
capacity. The program should be of moderate intensity and duration. Patients should be encouraged to exercise to
the point of pain, not through or beyond.

05 Sullivan A

You might also like